Modern India – UPSC 100 Questions

 

1.      With reference to the Provincial Government under Montford Reforms consider the following statements:

1. The Governor was the executive head of the province.

2. The Governor administered the reserved subjects through his executive council.

3. All members of provincial legislative councils were elected.

Which of the statements given above is/are correct?

(a) 1 and 2 only

(b) 2 and 3 only

(c) 1 and 3 only

(d) 1, 2 and 3

Answer: A

Montagu-Chelmsford Reforms and Government of India Act, 1919:

·        The British government, not prepared to part with or even share its power with the Indians, once again resorted to the policy of ‘carrot and stick. The carrot was represented by the insubstantial Montagu-Chelmsford Reforms, while measures such as the Rowlatt Act represented the stick.

·        In line with the government policy contained in Montagu’s statement of August 1917, the government announced further constitutional reforms in July 1918, known as Montagu-Chelmsford or Montford Reforms. Based on these, the Government of India Act, of 1919 was enacted.

·        Main Features: The main features of the Montford Reforms were as follows.

·        Provincial GovernmentIntroduction of Dyarchy: The Act introduced dyarchy for the executive at the provincial government level.

·        Executive: Dyarchy, i.e., the rule of twoexecutive councilors and famous ministerswas

introduced. The governor was to be the executive head of the province. Hence, statement 1 is correct.

·        Subjects were divided into two lists: ‘reserved’ which included subjects such as law and order, finance, land revenue, irrigation, etc., and ‘transferred’ subjects such as education, health, local government, industry, agriculture, excise, etc. The reserved subjects were to be administered by the governor through his executive council of bureaucrats, and the transferred subjects were to be administered by ministers nominated from among the elected members of the legislative council. Hence, statement 2 is correct.

·        The ministers were to be responsible to the legislature and had to resign if a no-confidence motion was passed against them by the legislature, while the executive councilors were not to be responsible to the legislature.

·        In case of failure of constitutional machinery in the province, the governor could take over the administration of transferred subjects also.

·        The secretary of state for India and the governor-general could interfere in respect of reserved subjects while in respect of transferred matters, the scope for their interference was restricted.

·        Legislature:

·        Provincial legislative councils were further expanded and 70 percent of the members were to be elected. Hence, statement 3 is not correct.

·        The system of communal and class electorates was further consolidated.

·        Women were also given the right to vote.

·        The legislative councils could initiate legislation but the governor’s assent was required. The governor could veto bills and issue ordinances.

·        The legislative councils could reject the budget but the governor could restore it, if necessary.

·        The legislators enjoyed the freedom of speech.

2.      In the context of literary contribution during the freedom struggle, Annie Besant was associated with which of the following literary works?

1. India: A Nation; A Plea for Self-Government

2. My path to atheism

3. Philosophy of Hinduism

Select the correct answer using the code given below.

(a) 1 only

(b) 1 and 2 only

(c) 2 and 3 only

(d) 1 and 3 only

Answer: B

Annie Besant was a prominent Theosophist, social reformer, political leader, women’s rights activist, writer

and orator. She was of Irish origin and made India her second home. She fought for the rights of Indians

and was the first woman president of the Indian National Congress.

Annie Besant’s famous literary works include

o India: A Nation; A Plea for Self-Government book was written by Annie Besant in 1915. Annie

Besant was known to be an untiring public worker as the head of various important movements and a

protagonist in the struggle for the extension of women’s rights.

In this book, she brought to the British eyes the actual Indian conditions and problems from the Indian point of view, and elucidated the true interests of India.

It sought to reconcile the Indian interests with the larger interests of the Empire by studying the political structure and with intimate and personal knowledge of Indian conditions and feelings.

o The Ancient Wisdom

o My path to atheism

o The Future Of Indian Politics

o A World Without God

o Gandhian Non – Co-operation; Or, Shall India Commit Suicide?

o Wake Up, India: A Plea For Social Reform

o A Study in Karma

As a member and later president of the Theosophical Society, Besant helped to spread Theosophical beliefs around the world, notably in India. Besant first visited India in 1893 and later settled there, becoming involved in the Indian nationalist movement.

In 1916 she established the Indian Home Rule League, of which she became president. She was also a leading member of the Indian National Congress. Her long-time interest in education resulted in

the founding of the Central Hindu College at Benares (1898).

Annie Besant founded two newspapers, “The Commonweal” and “New India.

Philosophy of Hinduism is one of the several unpublished writings of Dr. B.R Ambedkar.

Hence option (b) is the correct answer.

3.      Consider the following statements with respect to Mahatma Gandhi’s experience in South Africa:

1. Mahatma Gandhi started his weekly paper Indian Opinion as a means to serve and consolidate the Indian community.

2. During the Second Boer War, Gandhiji created Indian Ambulance Corps to help the British.

3. Gandhi-Smuts agreement was signed in 1905 to settle disputes regarding immigration.

Which of the statements given above are correct?

(a) 1 and 3 only

(b) 2 and 3 only

(c) 1 and 2 only

(d) 1, 2 and 3

Answer: C

Mahatma Gandhi, in 1898, went to South Africa in connection with a case involving his client, Dada

Abdullah. He witnessed the ugly face of white racism and the humiliation and contempt to which Asians were subjected. He stayed there till 1914 after which he returned to India.

During the early moderate phase, Gandhiji relied on sending petitions to authorities. In this phase, Gandhiji decided that the Indian Opinion (1903), his weekly paper, should be printed at a farm away from the city, he purchased an estate in Natal province, fourteen miles from Durban (South Africa). This would be a place where everyone would labor, and draw the same salary. He called this the Phoenix Settlement. Also, Gandhiji began Indian Opinion as a means to serve and consolidate the Indian community. The succession of meetings and events which culminated in the founding of the Phoenix Settlement began when Madanjit Vyavaharik (co-owner of Indian Opinion, with Gandhi) learnt of an outbreak of plague in the vicinity of Johannesburg while canvassing subscribers and collecting subscriptions. Hence statement 1 is correct.

In 1907 when the Transvaal legislature passed a law requiring all Asians to take out registration cards, he launched a campaign of passive resistance, coining the phrase, satyagraha.

He also set up the Tolstoy Farm (1910) for all those taking part in the movement. He also campaigned against restrictions on Indian migration and against poll tax.

The Indians protested the Transvaal Immigration Act, by illegally migrating from Natal into

Transvaal. Gokhale toured the whole country (India) mobilizing public opinion in support of the Indians in South Africa. Even Lord Hardinge, condemned the repression by the British.

Eventually, through a series of negotiations involving Gandhi, Lord Hardinge, C.F. Andrews, and General Smuts, an agreement (Gandhi-Smuts Agreement) was reached in June 1914 by which the Government of South Africa conceded the major Indian demands relating to the poll tax, the registration certificates and marriages solemnized according to Indian rites, and promised to treat the issue of Indian immigration in a sympathetic manner. Hence statement 3 is not correct.

The Natal Indian Ambulance Corps was created by Mahatma Gandhi for use by the British as stretcher bearers during the Second Boer War, with expenses met by the local Indian community. Gandhi was bestowed with the ‘Kaiser-i-Hind‘ and other medals by the British for his work in the Boer war. This was given up by Gandhi after the Jallianwala Bagh massacre. Hence statement 2 is correct.

 

4.      With reference to the Nagpur session of the Indian National Congress of 1920, consider the following statements:

1. C.R.Das moved the main resolution on non-cooperation.

2. Congress set the goal of the attainment of self-government through constitutional means.

3. Congress from now onwards decided to form provincial congress committees on a linguistic basis.

Which of the statements given above are correct?

(a) 1, 2 and 3

(b) 2 and 3 only

(c) 1 and 2 only

(d) 1 and 3 only

Answer: D

Nagpur session of the Indian National Congress, December 1920 :

o C.R. Das moved the main resolution on Non-Cooperation in the annual session of Congress in Nagpur in 1920. In the Nagpur session, Congress decided to the attainment of Swaraj through peaceful and legitimate means, thus committing itself to an extra-constitutional mass struggle. Hence statement 1 is correct.

o The program of non-cooperation was endorsed.

o An important change was made in the Congress creed: now, instead of having the attainment of self-government through constitutional means as its goal, Congress decided to have the attainment of swaraj through peaceful and legitimate means, thus committing itself to an extra-constitutional mass struggle. Hence, statement 2 is not correct.

o Some important organizational changes were made: a congress working committee (CWC) of 15

members was set up to lead the Congress from now onwards; provincial congress committees on a

linguistic basis were organized; Hence statement 3 is correct.

o Ward committees were organized, and the entry fee was reduced to four annas.

5.      In the context of revolutionary activities during the freedom struggle, who among the following was associated with the Berlin Committee for Indian Independence?

1. Virendranath Chattopadhyay

2. Bhupendranath Dutta

3. Lala Hardayal

Select the correct answer using the code given below.

(a) 1 only

(b) 1 and 2 only

(c) 2 and 3 only

(d) 1, 2 and 3

Answer: D

The first world war animated the revolutionaries with the hope that a great opportunity had come for

throwing off the shackles of the country’s subjugation. It created great revolutionary fervor. It was the belief of the revolutionaries that Great Britain disturbed by war would not be able to checkmate an armed uprising in India.

Rash Behari Bose alone contemplated the ‘All-India armed uprising’. In April 1914, Virendranath

Chattopadhyaya shifted himself to Germany where he was carrying on revolutionary activities in close association with Madam Kama, S.R. Rana (who were in Paris), and Jnanendra Dasgupta, Naren Sen, and Padmanabhan Pillai were in Switzerland.

The revolutionary leader’s Champak Raman Pillai and Chanji Kersump were already in Germany. The first initiative for taking German assistance for Indian revolutionaries was taken by Virendranath Chattopadhyaya and Abinash Bhattacharya.

They met Baron Oppenheim in Berlin in the first week of September 1914 and gave him a draft of the terms of mutual cooperation between the Indian revolutionaries and Germany. Oppenheim did a fine job and formed a society named Deutsche Verjin der Freunde Indian (Indian Friendship Society of Germany).

o Herr Albert Berlin (who was the then General Manager of Hamburg America Steamer Company) was its President. Baron Oppenheim and Dr. Sukhtankar were its Vice-Presidents and Dhiren Sarkar was its Secretary.

o The Society subsequently came to be known as the Indian Independence Committee and more popularly known as the Berlin Committee for Indian Independence. Virendranath

Chattopadhyaya was its Secretary till the end of 1916 when Bhupendra Nath Duta succeeded him.

Virendranath Chattopadhyaya sent Dhiren Sarkar and S.S. Martha to America to establish contact with the Indian revolutionaries living in America: Lala Har Dayal, Bhupendra Nath Dutta, Tarak Nath Das, Birendra Nath Dasgupta, etc. All came to Berlin and joined the Berlin Committee for Indian Independence.

Different plans were made by Berlin Committee for Indian Independence and the German Government for organizing an armed uprising in India during the first global war. There were three different schemes which are as follows:

o The Afghan Scheme: was directed towards utilizing the Muslim tribal disaffection in the NorthWest Frontier and establishing a provisional government.

o The Batavia Scheme: mainly relied on Bengal revolutionaries.

o The Bangkok Scheme: relied on the coordinated activities of revolutionaries in India and the ‘returned Sikhs’ of the Ghadar Party.

The Afghan Scheme was directed towards utilizing the Muslim tribal disaffection in the NorthWest Frontier and establishing a provisional government.

Hence option (d) is the correct answer.

6.      In the context of the freedom struggle, consider the following statements regarding Ghadar Movement:

1. Its objective was the establishment of an independent republic of India.

2. Bande Mataram was adopted as the rallying cry of the Ghadar movement.

3. Ghadarites criticized the loyalist role played by the Punjabis during 1857.

Which of the statements given above is/are correct?

(a) 1 only

(b) 1 and 2 only

(c) 2 and 3 only

(d) 1, 2 and 3

Answer: D

Ghadar means ‘revolt’ or rebellion. The Ghadar party (started in 1913) was a revolutionary group organized to overthrow British rule in India. It was organized by overseas Indian immigrants to Canada and the USA. It was clearly stated by the Ghadarites that their objective was the establishment of the independent republic of India. Hence statement 1 is correct.

o The party was organized around a weekly newspaper ‘The Ghadar’ which was published from its headquarters, the Yugantar Ashram in San Francisco.

o The founding president of the Ghadar party was Sohan Singh Bhakna and Lala Hardayal was a co-founder of this party.

o The leadership also included Bhagwan Singh, Barkatullah, and Ram Chandra. The Ghadar militants immediately began an extensive propaganda campaign against British rule.

o They toured extensively, visiting mills and farms where most of the Punjabi immigrant labour worked.

The Ghadarites were secular in outlook. The nationalist salute ‘Bande Mataram’ (and not any Sikh religious greeting ) was urged upon and adopted as the rallying cry of the Ghadar Movement. Hence statement 2 is correct.

o The Ghadarites sought to give a new meaning to religion as well. They urged that religion lay not in observing the outward forms such as those signified by long hair and Kirpan (sword), but in remaining true to the model of good behavior that was enjoined by all religious teachings.

Ghadarites did not betray any narrow regional loyalties. Lokamanya Tilak, Aurobindo Ghose, Khudi Ram Bose, Kanhia Lal Dutt, and Savarkar were all the heroes of the Ghadars. Rash Behari Bose was

importuned and accepted as the leader of the abortive Ghadar revolt in 1915.

Far from dwelling on the greatness of the Sikhs or the Punjabis, the Ghadars constantly criticized the loyalist role played by the Punjabis during 1857. Hence statement 3 is correct.

7.      Arrange the following events regarding Gandhiji’s struggle in South Africa in the correct chronological sequence:

1. Public burning of Registration Certificates

2. Formation of Passive Resistance Association.

3. Resistance against the imposition of the poll tax on all ex-indentured Indians.

Select the correct answer using the code given below

(a) 2-1-3

(b) 2-3-1

(c) 1-3-2

(d) 3-1-2

Answer: A

Gandhiji’s struggle in South Africa:

o Satyagraha against Registration Certificates (1906)

The government in South Africa enacted legislation making it compulsory for Indians to take out

certificates of registration that held their fingerprints. It was essential to carry these in person

at all times. At a huge public meeting held on 11 September 1906, in the Empire Theatre in

Johannesburg, Indians resolved that they would refuse to submit to this law and would face the

consequences. The Government remained adamant, and so did the Indians. Gandhiji formed the

Passive Resistance Association in 1906 to conduct the campaign.

General Smuts called Gandhiji for talks and promised to withdraw the legislation if Indians

voluntarily agreed to register themselves. Gandhiji accepted and was the first to register. But Smuts

ordered that the voluntary registrations be ratified under the law. The Indians under the leadership

of Gandhiji retaliated by publicly burning their registration certificates on 16, August 1908.

o Campaign against Poll Tax and Invalidation of Indian Marriages

The movement was widened further to include resistance to the poll tax of three pounds that

was imposed on all ex-indentured Indians. The inclusion of the demand for the abolition of this

tax, a particularly heavy charge on poor laborers whose wages hardly averaged ten shillings a

month, immediately drew the indentured and ex-indentured laborers into the struggle.

In March 1913, the satyagrahi’s campaign was motivated again by government action.

Supreme Court invalidated all marriages not conducted according to Christian rites and

registered by the Registrar of Marriages. By implication, Hindu, Muslim, and Parsi marriages

were illegal and the children born through these marriages were illegitimate.

 

8.      For which of the following purposes was the Passive Resistance Association formed by Gandhiji?

(a) To raise awareness about the passive resistance methods among Indian masses before participating in mainstream Indian politics.

(b) To conduct a Satyagraha campaign against mandatory carrying of registration certificates by migrant Indians in South Africa.

(c) To peacefully mobilize Ahmedabad Mill workers for fulfilling their demand of plague bonus.

(d) To advocate non-violent means of struggle among the militant nationalists.

Answer: B

During his stay in South Africa, Gandhi Ji took various initiatives for the betterment of discriminated

communities.

One such discrimination was imposed by new legislation in South Africa in 1906 which mandated migrant Indians to compulsorily carry at all times certificates of registration with their fingerprints.

The Indians under Gandhi’s leadership decided not to submit to this discriminatory measure.

Gandhi formed the Passive Resistance Association to conduct the Satyagraha campaign.

As a response, the government jailed Gandhi Ji and others who refused to register themselves. Later, the authorities used deceit to make these defiant Indians register themselves. The Indians under the leadership of Gandhi retaliated by publicly burning their registration certificates.

Hence option (b) is the correct answer.

9.      In the context of the growth of revolutionary nationalism during the British time, arrange the events in chronological order.

1. Unsuccessful attempt under the leadership of Rash Behari Bose and Sachin Sanyal to kill the Viceroy, Lord Hardinge.

2. Prafulla Chaki and Khudiram Bose threw a bomb at a carriage of Kingsford.

3. Assassination of Curzon-Wylie in London by Madan Lal Dhingra.

4. Chapekar brothers, Damodar and Bal Krishna murdered the plague commissioner of Poona.

Select the correct answer using the code given below.

(a) 2-3-4-1

(b) 4-3-2-1

(c) 2-4-3-1

(d) 4-2-3-1

Answer: D

Government repression and frustration caused by the failure of the leadership to provide a positive lead to the people ultimately resulted in revolutionary terrorism, The youth of Bengal found all avenues of peaceful protest and political action blocked and out of desperation they fell back upon individual heroic action and the cult of the bomb.

Yugantar wrote on 22 April, 1906 after the Barisal Conference: “The remedy lies with the people themselves. The 30 crores of people inhabiting India must raise their 60 crores of hands to stop this curse of oppression. Force must be stopped by force”.

Revolutionary Activities

Bengal

o 1902First revolutionary groups in Midnapore and Calcutta (The Anushilan Samiti)

o 1907Attempt on life of the former Lt. governor of East Bengal and Assam.

o 1908Prafulla Chaki and Khudiram Bose attempt to murder Muzaffarpur Magistrate,

Kingsford.

o 1908Burrah dacoity by Dacca Anushilan.

o 1912Bomb thrown at Viceroy Hardinge by Rashbehari Bose and Sachin Sanyal.

Maharashtra

o 1879Ramosi Peasant Force by Vasudev Balwant Phadke.

o 1890s—Tilak’s attempts to propagate militancy among the youth through Shivaji and Ganapati

festivals, and his journals Kesari and Maharatta.

o 1897Chapekar brothers kill Rand, the plague commissioner of Poona and Lt. Ayerst.

o 1899Mitra Melaa secret society organised by Savarkar and his brother.

o 1904Mitra Mela merged with Abhinav Bharat.

o 1909District Magistrate of NasikJacksonkilled.

Revolutionary Activity Abroad

o 1905Shyamji Krishnavarma set up Indian Home Rule Society and India House and brought out

journal The Sociologist in London.

o 1909Madan Lal Dhingra murdered Curzon-Wyllie; Madame Bhikaji Cama operated from Paris and Geneva and brought out journal Bande Mataram.

Hence option (d) is the correct answer.

10.  In the context of the freedom struggle, who among the following belonged to the extremist faction of the Indian National Congress?

1. Rahmat Ullah Sayani

2. Bipin Chandra Pal

3. Anand Mohan Bose

Select the correct answer using the code given below.

(a) 1 and 3 only

(b) 2 only

(c) 2 and 3 only

(d) 1 and 2 only

Answer: B

The extremist period of the Indian national movement broadly starts from 1905 -1919. This period of the Indian national movement witnessed a militant nationalist approach to political activities. Their mode of struggle was

o Passive resistance

o Mass Agitation

o Self-reliance

Some of the prominent leaders of the extremist phase of the national movement are:

o Aurobindo Ghosh

o Ashwini Kumar Dutt

o Lala Lajpat Rai

o Bal Gangadhar Tilak

o Bipin Chandra Pal

o Brahma Bandhav Upadhyay

o Satish Chandra Mukherjee

o Pulin Das

o Ras Bihari Bose

o Apurva Kumar Ghosh

o Rajnikant Sen

Rahmat Ullah Sayani was born on 5 April, 1847 in Bombay. He was one of the two famous Muslim representatives, who attended the first session of the Indian National Congress in 1885, which was attended by 72 members only.

o Later, he became the Mayor of the Bombay Municipal Corporation in 1888 and, in the same year, became a Member of Bombay Legislative Council. He continued in the Bombay Legislative Council and was in that capacity till 1896.

o Rahmatullah Mohammad Sayani also became a member of the Imperial Legislative Council in

1896. He worked hard for the promotion of religious harmony and peace among different sections of

society. He belonged to the moderate faction of congress

o When Sir Syed Ahmed Khan was campaigning against the Indian National Congress, he powerfully countered the campaign.

Ananda Mohan Bose was born on 23 September 1847 in Mymensingh, Bengal in an upper-middle-class family. He was a leader of the Brahmo Samaj, pioneer of the freedom movement, an educationist, and a social reformer.

o Bose’s interest in the political scene in India may be dated from 1871 when he first met Surendranath Banerjea in England. On his return to India in 1874 and right up to the days of the Swadeshi movement in 1905, the two were closely associated in all their political enterprises.

o With Surendranath as his mentor and through his own organizational ability, Ananda Mohan Bose set up a number of pioneering institutions. The Calcutta Students Association was the earliest attempt made to organize students for constructive political work. The Indian Association was the first political organization at an All – India level to institute a vigorous constitutional agitation for the rights and privileges of the Indian citizens.

o One of its by-products was the convening of the, first National Conference in 1883 which became a precursor of the Indian National Congress (1885).

o Bose was associated with the Congress since its inauguration and was elected President of its

Madras Session in 1898.

o Under his enlightened direction, the Sadharan Brahmo Samaj, of which he was a joint founder

(1878), became an active centre for the spread of education and social uplift.

o A moderate and a constitutionalist in his political outlook, Bose was progressive and one of the earliest to have pleaded for large scale technical education and industrialization.

o He is remembered in particular for the last speech that he made on 16 October 1905 at a public meeting organised in Calcutta to protest against the partition of Bengal.

Bipin Chandra Pal was born on 7th November 1858 in Poil village in Sylhet district in present-day Bangladesh. Bipin Chandra Pal is known as the ‘Father of Revolutionary Thoughts’ in India. He was also an eminent radical of his time.

o Bipin Chandra Pal used his profession of journalism in spreading patriotic awareness. He published a number of journals, weekly and books to spread Swaraj. His prominent books include ‘Nationality and Empire’, ‘Indian nationalism’ , ‘Swaraj and the Present Situation, ‘The Soul of India’, ‘The Basis of Social Reform’, ‘The Hinduism’ and ‘The New Spirit’.

o He was also the editor of the ‘Democrat’, the ‘Independent’ and many other journals. He also started journals like ‘Paridarsak’, ‘New India’, ‘Bande Mataram’ and ‘Swaraj’.

o Bipin Chandra Pal was famous as one of the triumvirates of the three militant patriots popularly known as ‘Lal-Bal-Pal’. These three was responsible for initiating the first popular upsurge against the British colonial policy in the 1905 partition of Bengal.

o At the time of Bal Gangadhar Tilak’s arrest and government repression in 1907. He left for England where he was briefly associated with the radical India House and also founded the Swaraj journal.

Hence option (b) is the correct answer.

11.  With reference to the Indian National Congress session of the year 1905, consider the following statements:

1. It was held in Calcutta.

2. The formal proclamation of the Swadeshi movement took place in this session.

3. At the time of this session, Lord Hardinge was the viceroy of British India.

Which of the statements given above is/are not correct?

(a) 1 only

(b) 1 and 2 only

(c) 2 and 3 only

(d) 1, 2 and 3

Answer: D

The 21st session of the Indian National Congress was held at Benares from Dec. 27-30, 1905. Gopal Krishna Gokhale was the president of this session. Hence statement 1 is not correct.

The formal proclamation of the Swadeshi Movement took place on 7th August 1905 in a meeting held at the Calcutta Town Hall. The Boycott movement was also launched along with the Swadeshi movement. Hence statement 2 is not correct.

The division between extremists and moderates could also be felt for the first time in this session.

Lord Minto was the Viceroy of India from 1905 -1910. Lord Hardinge was the Viceroy of India from 191016. (Lord Curzon resigned in the year 1905 itself after which Lord Minto was appointed in the same year). Hence statement 3 is not correct.

 

12.  With reference to the infamous 1908 sedition case against Bal Gangadhar Tilak, consider the following statements:

1. Tilak was charged with sedition for defending revolutionaries Khudiram Bose and Prafulla Chaki in his newspaper Kesari.

2. Mohammed Ali Jinnah was one of the lawyers who defended Tilak in court.

3. At the end of the trial Tilak was acquitted of all charges.

Which of the statements given above is/are correct?

(a) 1 only

(b) 1 and 2 only

(c) 2 and 3 only

(d) 1, 2 and 3

Answer: B

During his lifetime, Bal Gangadhar Tilak had been tried for Sedition Charges three times by the British rulers in 1897, 1909, and 1916.

On 30 April 1908, two youths, Prafulla Chaki and Khudiram Bose threw a bomb on a carriage at

Muzzafarpur, to kill the Chief Presidency Magistrate Douglas Kingsford of Calcutta, but erroneously killed two women traveling in it. While Chaki committed suicide when caught, Bose was hanged. Tilak, in his paper Kesari, defended the revolutionaries and called for immediate Swaraj or self-rule. The Government swiftly charged him with sedition. Hence statement 1 is correct.

On 24 June 1908, Tilak was arrested from Bombay on a charge of sedition and 153A IPC in respect of two articles carried in Kesari on 12 May and 9 June 1908. The article dated 12 May was titled ‘The Country’s Misfortune’ and that of 9 June has titled ‘These Remedies Are Not Lasting’. His house in Poona was searched by the police that found a postcard with the names of two books on explosives written on it. The Chief Presidency Magistrate of Bombay declined an application for bail. The sanction for the prosecution was swiftly granted and Tilak was committed to stand trial at the sessions of the Bombay High Court.

Tilak was defended by Mohammed Ali Jinnah, later by Joseph Baptista, and later by himself assisted by a battery of counsel. Hence statement 2 is correct.

The special jury comprised seven Europeans and two Indians. Indian jurors returned a verdict of not guilty while the European jurors adjudged Tilak guilty.

Justice Davar awarded Tilak a sentence of six years’ transportation and after some time Tilak was sent to a prison in Mandalay in Burma. Hence statement 3 is not correct.

 

13.  With reference to the ‘Ahmedabad mill strike’, consider the following statements:

1. The primary reason for the strike was the discontinuation of the plague bonus given in times of crisis to the workers.

2. It was Gandhiji’s first hunger strike in the Indian National Movement.

Which of the statements given above is/are correct?

(a) 1 only

(b) 2 only

(c) Both 1 and 2

(d) Neither 1 nor 2

Answer: C

About the Ahmedabad Mill strike:

o In 1917 plague outbreak made the labor shortage more acute because it drove many workers away from Ahmedabad to the countryside. To dissuade the workers from leaving the town, the mill owners decided to pay the plague bonus which was sometimes as high as 75% of the normal wages of the workers.

o After the epidemic was over, the mill owners decided to discontinue the Plague Bonus. But the workers opposed the employers’ move and argued that it was helping them to offset the wartime rise in the cost of living. The mill owners were prepared to give a 20% increase but the workers were demanding a 50% raise in wages in view of the price hike. Hence statement 1 is correct

o Gandhiji began the Satyagraha movement against the mill owners. The workers were asked to take a pledge stating that they would not resume work without a 35% increase and that they would remain

law-abiding during the lockout.

o Gandhiji, assisted by Anasuya Sarabhai organized daily mass meetings of workers, in which he

delivered lectures and issued a series of leaflets on the situation. It was Gandhi’s first Hunger strike in

the Gandhian movements in India. Hence statement 2 is correct

o Finally, the arbitrator’s award went in favor of the workers and 35% raise was given to them.

14.  In the context of the growth of communalism during the freedom struggle, consider the following statements regarding the Shimla Deputation of 1906:

1. It was led by Muhammad Ali Jinnah.

2. It was in this deputation that the claim to separate Muslim representation was made to the Viceroy.

3. In this deputation it was proposed that Muslim representation in the Imperial Legislative Council should be determined on the basis of numerical strength.

Which of the statements given above is/are not correct?

(a) 1 only

(b) 1 and 3 only

(c) 3 only

(d) 1, 2 and 3

Answer: B

On 1st October 1906, thirty-five Muslim leaders from all over India gathered in the viceregal Lodge at Shimla to present a memorandum to Lord Minto. They were led by Agha Khan, a former member of the Viceroy’s Legislative Council. ‘It was in this address that the claim to separate Muslim representation was made, and it was in the reply given by His Excellency that the claim is commonly understood to have been conceded. Hence statement 1 is not correct and statement 2 is correct.

They stated that Muslims are a distinct community with additional interests of their own, which are not shared by other communities, and no Muslim would ever be returned by the existing electoral bodies unless he worked, in sympathy with the Hindu majority in all matters of importance.

On these grounds the deputation made the following proposals :

o In the case of municipal and district boards, the number of Hindus and Muslims entitled to scats should be declared; such proportion being determined in accordance with the numerical strength, social status, local influence, and special requirements of either community and that each community should be their own representative, as in the Aligarh Municipality and in many towns in Punjab.

o In the case of provincial Legislative Councils, the proportion of Muslim representatives should be

determined and declared with due regard to the considerations noted above, and that the important

Muslim landlords, lawyers, merchants, and representatives of other important interests, the Muslim

members of district boards and municipalities and Muslim graduates of universities of a certain

standing, say five years, should be formed into electoral colleges and he authorized to return the number of members that may be declared to the eligible.

o For their representation in the Imperial Legislative Council, they suggested:-

That the proportion of Muslims should not be determined on the basis of numerical strength

and that they should never be an ineffective minority.

Thus the memorial pointed out, “the position accorded to the Muslim Community in any kind

of representation, direct or indirect, and in all other ways affecting their status and influence

should be commensurate not merely with their numerical strength, but also with their

political importance and the value of the contribution which they make to the defense of the

Empire. Hence statement 3 is not correct.

That as far as possible appointment by-election should be given preference over the nomination.

For the purpose of choosing Muslim members, Muslim landowners, lawyers and merchants, and

representatives of every important interest of status to be subsequently determined by the

government, Muslim members of provincial legislative councils and Muslim fellows of universities

should be invested with electoral powers.

15.  With reference to the Home Rule Movement during the freedom struggle, consider the following statements :

1. Joseph Baptista was the president of Tilak’s Home Rule League.

2. Deputations of both Bal Gangadhar Tilak’s and Annie Besant’s Leagues pleaded for the early grant of responsible government before the Joint Parliamentary Committee in Britain.

3. At the Lucknow session of the Indian National Congress (1916), Tilak’s Home Rule League was allowed to propagate the message of the self-government resolution of congress.

Which of the statements given above is/are correct?

(a) 1 only

(b) 1 and 2 only

(c) 2 and 3 only

(d) 1, 2 and 3

 

Answer: D

The last phase of Tilak’s career, 1914-20, was marked by his sponsorship of the Indian Home Rule League. It may be noted that until the Lucknow session of the Indian National Congress in December 1916 Tilak was not readmitted to its sessions and committees meetings.

o The branches of the All India Home Rule League of Annie Besant were afforded the opportunity of propagating the message of the Self-Government resolution of Congress. But even at the Lucknow session of the Congress, Tilak’s Indian Home Rule League and its branches were not afforded this opportunity. Hence statement 3 is correct.

Tilak established the Indian Home Rule League on 28 April 1916 at Belgaum, while Annie Besant

commenced propaganda of her Home Rule League earlier in January 1916.

The President of Tilak’s Home Rule League was Joseph Baptista of Bombay and Secretaries N.C. Kelkar and Gokhale. Hence statement 1 is correct.

Within a year since its establishment, the membership of Tilak’s League registered strength of thirteen to fourteen thousand; in 1918 it reached thirty-four thousand. The League’s membership was drawn from people of different religions, including Muslims and Christians, of different castes among Hindus, upper, middle, and lower, and also from women. Annie Besant’s League’s appeal was mainly directed at the English educated.

The deputations of both Tilak’s and Besant’s Leagues went to Britain independently of that of the Congress to plead for the early grant of responsible government before the Joint Parliamentary Committee. Hence statement 2 is correct.

The Home Rule League movement not only afforded Tilak and his followers an independent platform apart from Congress, but it also consolidated his own following, and most importantly, exerted pressure on Congress to own the demand for home rule and transmit it to the British government both here and in Britain.

 

16.  Which of the following were the achievements of Swarajists?

1. Formation of a clear majority government in the provincial assembly of Central Provinces in 1923.

2. Election of Vithalbhai Patel as the speaker of the Central Legislative Assembly in 1925.

3. The defeat of the Public Safety Bill in 1928.

Select the correct answer using the code given below.

(a) 1, 2 and 3

(b) 2 and 3 only

(c) 1 and 3 only

(d) 1 and 2 only

 

Answer: A

o Achievements

With coalition partners, they out-voted the government several times, even on matters relating to

budgetary grants, and passed adjournment motions.

They agitated through powerful speeches on self-government, civil liberties, and industrialization.

Vithalbhai Patel was elected speaker of the Central Legislative Assembly in 1925. Hence

statement 2 is correct.

A noteworthy achievement was the defeat of the Public Safety Bill in 1928, which was aimed at

empowering the Government to deport undesirable and subversive foreigners (because the

Government was alarmed by the spread of socialist and communist ideas and believed that a crucial

role was being played by the British and other foreign activists being sent by the Comintern). Hence

statement 3 is correct.

Through their activities, they filled the political vacuum at a time when the national movement was

recouping its strength.

They exposed the hollowness of the Montford scheme.

They demonstrated that the councils could be used creatively.

NOTE: In the November 1923 elections, the Swarajists had managed to win 42 out of 141

elected seats and a clear majority in the provincial assembly of Central Provinces. In

legislatures, in cooperation with the Liberals and the independents like Jinnah and Malaviya, they

won a majority. Hence statement 1 is correct.

17.  Consider the following statements regarding the Delhi Durbar of 1911:

1. It was held to mark the coronation of King George V.

2. All the ceremonies, traditions, and processions associated with the durbar were exclusively British and no Indian element was involved.

Which of the statements given above is/are correct?

(a) 1 only

(b) 2 only

(c) Both 1 and 2

(d) Neither 1 nor 2

Answer: A

The Darbar of 1911 marked the succession of King George V. This Durbar was historic for two

reasons; one it was the only one attended by the Emperor himself and second, this was where the shifting of the Imperial Capital from Calcutta to Delhi was announced. Hence statement 1 is correct.

The Proclamation Ceremony took place on 12th December in the Durbar arena which had two amphitheatres

on the same site as the previous Durbars. The larger, outer one called the Spectator’s Mound was for the general public. The other smaller amphitheatre was reserved for the esteemed guests of the Durbar.

This Proclamation Ceremony of 1911 was much talked about for various reasons. Firstly, the

Maharaja of Gaekwar after arriving at the amphitheatre removed all his jewellery except his Star and medals and handed them over to his son who was sitting behind him. This behaviour of the Maharaja was considered rude.

o Secondly, the planners of the Darbar had some idea long before the Ceremony that few Ruling Chiefs were not in favour of bowing before the Emperor for the Homage Ceremony. They were expected to bow three times before their Imperial Majesties and walk back to their places in reverse. They wanted to meet the Emperor but as equals.

o Hence, on the day of the Ceremony, Maharaja of Gaekwar during his homage to the King and the Queen partly bowed, just once, turned around and walked to his place. His behaviour at the Delhi Durbar was written about in many newspapers as it was considered highly disrespectful on his part. He later apologized on the advice of the British Resident in his court.

o It soon came to be known as the ‘Durbar Incident. Reactions were swift and ranged all the way from calls for the Maharaja’s dismissal to praise and the attempt to use the incident as a nationalist political rallying point against British rule.

The planners of the Royal Durbar, led by Sir John Hewett, completed the symbolic relocation of the event by incorporating South Asian motifs throughout the design. The architect of the Durbar arena, Sir Samuel Swinton Jacob, developed these motifs after touring the remains of important Indian architectural sites, including Fatehpur Sikri, Agra, and Delhi. He eventually produced an amalgam, combining (in his view) the very best of ancient Hindu and Islamic art. He called this ensemble “Indo- Saracenic.

o Ceremonially, too, the planners tried to combine and incorporate the elements of as many Indian imperial traditions as possible. When the King-Emperor arrived, for example, his progress to the Durbar site was along the Moghal via sacra, from the Red Fort’s Delhi Gate, across the central

plaissance, to the Jama Masjid, the largest mosque in South Asia. This was the route the Moghal

emperors had used to attend Friday evening prayers. Hence statement 2 is not correct.

18.  In the context of the freedom struggle, the firebrand leader Lala Lajpat Rai was associated with which of the following organizations?

1. Social Service League

2. Home Rule League of America

3. Servants of the People Society

Select the correct answer using the code given below.

(a) 1 only

(b) 1 and 2 only

(c) 2 and 3 only

(d) 1, 2 and 3

Answer: C

Lala Lajpat Rai was a prominent nationalist leader who played an important role in India’s struggle for freedom. His fierce brand of patriotism and potent vocalism against British rule earned him the title of ‘Punjab Kesari’ or the Lion of Punjab.

He, together with other prominent leaders like Bipin Chandra Pal, Aurobindo Ghosh from Bengal, and Bal Gangadhar Tilak from Maharashtra, began to see the negative aspects of moderate politics advocated by many leaders of the Indian National Congress. They voiced their strong opposition to the Congress’ demand for gradual progress to dominion status and began voicing the need for complete independence or ‘Purna Swaraj.

Lala Lajpat Rai gave up his legal practice and put all his efforts into freeing his Motherland from the

shackles of British Imperialism.

o He went to Britain in 1914 and then to the USA in 1917. In October 1917, he founded the Indian

Home Rule League of America in New York. He stayed in the USA from 1917 to 1920.

In 1920, after his return from America, Lajpat Rai was invited to preside over the special session of the Congress in Calcutta. He led fiery demonstrations against the British in Punjab in protest of their brutal actions at Jallianwallah Bagh.

o When Gandhi launched the non-cooperation movement in 1920, he plunged into action leading the

movement in Punjab. When Gandhi decided to suspend the movement post the Chauri Chaura incident, Lajpat Rai criticized the decision and went on to form the Congress Independence Party.

Servants Of the People Society was founded by Lala Lajpat Rai in 1921 at Lahore, and was

inaugurated by Mahatma Gandhi on 9th November 1921.

On October 30, 1928, Lala Lajpat Rai led a peaceful procession to oppose the arrival of the Simon

Commission in Lahore. Intercepting the march, Superintendent of Police, James A.Scott ordered his police force to ‘lathi-charge’ at the activists. The police targeted Lajpat Rai, in particular, and hit him in the chest.

This action left Lala Lajpat Rai with severe injuries. He died of a heart attack on November 17, 1928.

o His followers squarely placed the blame on the British and vowed to avenge his death. Chandrasekhar Azad along with Bhagat Singh and other associates plotted the assassination of Scott but the revolutionaries shot J.P. Saunders, mistaking him as Scott.

Lala Lajpat Rai’s publications include :

o The Story of My Deportation (1908)

o Arya Samaj (1915)

o The United States of America: A Hindu’s impressions (1916)

o Young India; an interpretation and a history of the nationalist movement from within (1916). Hence statement 3 is correct.

o England’s Debt to India: India (1917)

The Social Service League was established in 1911 by Narayan Malhar Joshi.

o It mainly worked for the improving condition of the common masses; opened schools, libraries etc.

o Narayan Malhar Joshi also known as Nana Saheb Joshi was born on 5 June 1879 at Goregaon, Kolaba district,. He co-founded the All India Trade Union Congress in 1920, was a member of the Bombay Provincial Congress Committee, and was a prominent member of the People’s Volunteer Brigade(P V B).

Hence option (c) is the correct answer.

 

 

 

 

19.  In the context of the Swadeshi movement, the Risley Circular in 1906 aimed at:

(a) financial decentralization of local bodies.

(b) prohibition of students’ participation in politics.

(c) regulating famines measures.

(d) extradition of nationalists from princely states.

Answer: B

Following the 1905 Partition of Bengal, (Banga-Bhang), the British regime let loose a cycle of

repression throughout the country. In view of the significant participation of students in political agitation, the government, in 1906, under a new circular ‘Risley Circular’ (named after H.H. Risley, the then Secretary to the Government of India, responsible for mooting the idea) prohibited students’ participation in politics or any other agitation.

The raising of slogans like ‘Vande Mataram and Tilak Maharaj Ki Jai was made a punishable offense. Earlier, the provincial government of Bengal had issued a ‘Carlyle Circular’; the Riley Circular was a modified version of the Carlyle Circular issued earlier in Bengal in October 1905.

Maharshi Aurobindo Ghosh, in his article ‘True Meaning of Risley Circular’ in the newspaper Vande

Mataram accused the imperial regime of trying to deprive the students of joining the patriotic stream,

sentiment, and programs. The regime’s true intent was to weaken the anti-imperialism agitation, Aurobindo asserted.

Hence option (b) is the correct answer.

20.  In the context of the freedom struggle, consider the following statements regarding Swadesh Bandhab Samiti:

1. It was set up by Ashwini Kumar Dutta.

2. Its main aim was to encourage the consumption of native products and boycott European goods.

Which of the statements given above is/are correct?

(a) 1 only

(b) 2 only

(c) Both 1 and 2

(d) Neither 1 nor 2

Answer: C

Corps of volunteers (or samitis as they were called) were a major form of mass mobilization widely used by the Swadeshi Movement. The Swadesh Bandhab Samiti set up by Ashwini Kumar Dutta, a school teacher, in Barisal was the most well-known volunteer organization of them all. Hence statement 1 is correct.

Its main aim was to encourage the consumption of native products and boycott European goods. The British Indian Government of India of the newly formed Eastern Bengal and Assam banned the Swadesh Bandhab Samiti in the year 1908. He was deported to the United Provinces where Dutta was imprisoned at the Lucknow jail. Hence statement 2 is correct.

Through the activities of this Samiti, whose 159 branches reached out to the remotest corners of the district, Dutt was able to generate an unparalleled mass following among the predominantly Muslim Peasantry of the region.

The samitis took the Swadeshi message to the villages through magic lantern lectures and Swadeshi songs, gave physical and moral training to the members, did social work during famines and epidemics, organized schools, training in Swadeshi craft and arbitration courts.

Ashwini Kumar Dutta, popularly known as Ashwini Dutta was a popular Indian freedom fighter,

philanthropist, educationist, social reformer, and nationalist.

o In 1887, Ashwini Kumar Dutta took the initiative to set up the District Board in Barisal. Dutta also

established the Bakarganj Hitaishini Sabha as well as a girls’ school in that year.

o In the year 1888, Dutta was appointed as the Vice Chairman of Barisal Municipality. Later in 1889, he constructed the Brojomohun College.

o Ashwini Kumar Dutta also undertook relief work after the cyclone in Barisal in 1919. At the Calcutta session of the Indian National Congress Party in the year 1921, he supported the non-violent Non- Cooperation Movement. Mohandas Gandhi, popularly known as Mahatma Gandhi, went in Barisal in the same year to respect the great leader. Ashwini Kumar Dutta also supported the workers of Assam Bengal Railway and Steamer Company who started a strike as a protest against the hostilities and violence on the workers of tea plantations of Assam in the year 1922.

o Ashwini Kumar Dutta also wrote several books on philosophy, religion and patriotism in Bengali

language.

o Some of his most well known books include, Bharatgeeti, Atmapratistha, Bhaktiyoga.

21.  Consider the following statements with respect to Foreign Cloth Boycott Committee:

1. It was formed by Indian National Congress during the Non-Cooperation movement.

2. Madan Mohan Malaviya was a member of the Committee.

3. Its objective was to bring about boycott of foreign cloth by advocating the adoption of handwoven Khadar.

Which of the statements given above is/are correct?

(a) 1 only

(b) 1, 2 and 3

(c) 3 only

(d) 2 and 3 only

Answer: D

The Working Committee of the Congress at its meeting in New Delhi on the 17th and 19th of

February 1929 appointed the following persons with power to co-opt, to constitute the Foreign Cloth

Boycott Committee:

o Mahatma Gandhi (Chairman)

o Motilal Nehru

o Madan Mohan Malaviya

o Moulana Abul Kalam Azad

o M.A Ansari

o Jawaharlal Nehru

The Non-Cooperation Movement was launched in 1920 and was called off in 1922. Hence statement 1 is not correct and statement 2 is correct.

The Committee was required to carry out the following item of the Congress programme:

o Inside and outside the legislatures methods suited to respective environments shall be immediately

adopted to bring about a boycott of foreign cloth by advocating and stimulating the production

and adoption of handspun and handwoven Khaddar.

o Hence statement 3 is correct.

The Foreign Cloth Boycott Committee approached all the Municipalities and District Local Boards in India

o To exempt Khadi from tax

o To increase the tax on foreign cloth and Purchase only Khadi for their use.

22.  Who among the following were the members of the Imperial Legislative Council?

1. Sachchidananda Sinha

2. Madan Mohan Malaviya

3. C. Vijayaraghavachariar

Select the correct answer using the code given below.

(a) 1 only

(b) 1 and 2 only

(c) 2 and 3 only

(d) 1, 2 and 3

Answer: D

Sachchidananda Sinha was born on 10th November 1871 in Arrah, in the erstwhile Bengal Presidency. In London, he was an active member of the British Committee on the Indian National Congress. He campaigned for the election of Dadabhai Naoroji to the House of Commons in 1892.

o In 1893, he enrolled himself at the Calcutta High Court and later practiced in Allahabad and Patna High Court.

o He played a crucial role in the formation of the Province of Bihar and Orissa. Sinha was associated with the Congress party between 1896 and 1919. During this time, he was an integral part of Bihar Provincial Congress Committee, serving as its Secretary and later as President.

o From 1910-1930, Sinha was a member of the Imperial Legislative Council. He became the Deputy President of the Central Legislative Assembly in 1921.

o He also held the office of the President in the Bihar and Orissa Legislative Council and served in the Bihar Legislative Assembly. Later, he became the first Indian ever to behold the portfolio of a Finance Member of a province.

o Sinha showed a keen interest in journalism and became the editor of the Hindustan Review, a monthly magazine.

Madan Mohan Malaviya, also called Mahamana was an Indian scholar, educational reformer, and a leader of the Indian nationalist movement.

o He made his political debut at the 1886 Calcutta (Kolkata) session of the Indian National

Congress. An industrious worker, he soon climbed the ranks in the party and was elected

president of Congress four times. Malaviya also served on the Imperial Legislative Council (190920).

o A gifted orator, he participated actively in debates on issues including free and compulsory primary

education, the prohibition on recruiting of Indian indentured labour in the British colonies, and the

nationalization of railways.

o Though a strong supporter of Congress, Malaviya helped establish the Hindu Mahasabha in 1906, which brought diverse local Hindu nationalist movements together.

o Malaviya, who was keenly interested in uplifting the educational standards of the country, was the

principal founder in 1916 of the Banaras Hindu University in Varanasi, a premier institution of learning in India.

o Malaviya’s consciousness toward the public led to the launch of his own Hindi-language weekly, the Abhyudaya (1907), the Leader of Allahabad, an English-language daily (1909), and the Hindi monthly the Maryada (1910).

Vijayaraghavachariar was born on 18 June 1852 in the village of Pon Vilaindha Kalathur, in the district of Chengalpattu in the state of Madras Presidency.

o After a successful career of a lawyer, he became a Member of the Legislature in Madras ( Chennai) in 1895 and he served the people until 1901. It was in Chennai that he came into close contact with

V.S.Srinivasa Sastri, C.Sankaran Nair, Sir V.Bhashyam Iyengar, and many other eminent citizens. He

ensured that the Chairpersons of the Municipalities were elected. A number of resolutions were passed due to his efforts. He was in the Imperial Legislature from 1913 for 3 years.

o He was involved with the Indian National Congress from day one. It was during the formative period that Vijayaraghavachariar and his friends played a key role in making the Indian National Congress an eff political organization with a focus on national consciousness, unity and development.

o He was appointed as a member of the Congress Propaganda Committee in 1899. It was at the

Calcutta Session in 1906 that he moved the resolution relating to the Permanent Land Settlement of Land Tenures.

o C. Vijayaraghavachariar believed in gender equality and wanted women to be of the right age at the

time of marriage. He was a classical liberal who believed in pragmatism.

Hence option (d) is the correct answer.

23.  With reference to the Indian Social Conference, consider the following statements:

1. It was founded by M.G. Ranade and Raghunath Rao.

2. Its first session was held in Bombay in December 1885 along with the first session of the Indian National Congress.

3. It launched the famous ‘Pledge Movement’ to inspire people to take an oath to prohibit child marriage.

Which of the statements given above is/are correct?

(a) 2 only

(b) 2 and 3 only

(c) 1 and 3 only

(d) 1, 2 and 3

Answer: C

When the Indian National Congress (INC) was formed in 1885, it was led by ardent social reformers such as M.G.Ranade and R.Ragunath Rao who were under the impression that the Congress would be concerned with social, economic, and political issues. However, it soon became clear that a strong faction in Congress was opposed to including social reform as part of the procedure.

The gist of the argument was that social reform was divisive and contentious, whereas Congress’s goal was political unity. As a result, social issues were omitted. M.G.Ranade and R.Ragunath Rao’s response to the INC’s argument was the formation of the Indian Social Conference. Hence statement 1 is correct.

Indian Social Conference met for the first time in Madras in 1887. M.G.Ranade’s clear intention was to bind the social reform movement as closely as possible to Congress, even if it remained organizationally separate. It acts virtually as the social reform cell of the Indian National Congress. Later Ranade arranged sessions of the conference every year, immediately following the session of Congress, and at the same location. Hence statement 2 is not correct.

The Conference’s goal was to bring together representatives of progressive Indian opinion on social issues, much like the Indian National Congress did for politics.

The Conference’s mission was to stimulate and strengthen the forces of reform by bringing together every year in mutual consultation representatives of the various associations and movements that, scattered across India, were struggling with social evils.

The Conference advocated intercaste marriages and opposed kulinism and polygamy. It launched the famous “Pledge Movement” to inspire people to take an oath to prohibit child marriage. Hence statement 3 is correct.

 

24.  He was born in the Kaya village in the Kushtia district of undivided Bengal. As a college student, he participated in the relief work undertaken by Ramakrishna Mission, on the streets of cholera-hit Calcutta. He was tasked with creating a “network of the secret society” to train dedicated youth for the revolution against the British by Aurobindo Ghosh, later known as Jugantar. He was arrested in the Alipore bomb case and the Howrah conspiracy case. In 1912, he met the German Crown Prince in Kolkata and asked him for arms for an insurrection to create a socialist government in India.

Who among the following is being described in the passage given above?

(a) Jatindranath Mukherjee

(b) Barindra Kumar Ghosh

(c) Sachindra Nath Sanyal

(d) Rash Behari Bose

Answer: A

Born in Kaya village in the Kushtia district of the undivided Bengal, part of present-day Bangladesh, in 1879, Jatindranath Mukherjee kindled the flame of revolution against the British colonial rule in the Indian subcontinent.

Jatin envisioned a modern India politically free, economically prosperous, and spiritually progressive.

The epithet ‘Bagha Jatin’ was earned by young Jatindranath Mukherjee in 1906 when he fought with a Royal Bengal tiger alone for three hours and killed it with a dagger.

During his teenage, Jatin was profoundly influenced by the Bhagavad Gita and the writings of Bankim Chandra Chattopadhyay. After completing school, Jatin enrolled in Central College of Calcutta. As a college student, Jatin participated in the relief work undertaken by the Ramakrishna Mission, on the streets of cholera-hit Calcutta. There he came in contact with Sister Nivedita, the Irish disciple of Swami Vivekananda.

Jatindranath Mukherjee’s meeting with Sri Aurobindo ignited his fervor for revolution against the British Raj. Soon Aurobindo considered Jatin as his right-hand man. It was Sri Aurobindo who entrusted Jatin with the crucial task of creating a “network of the secret society” to train dedicated youth for the revolution against the British. That secret society came to be known as Jugantar, and Bagha Jatin became its commander-in-chief.

The nation was seething with discontent against the British Raj. It was at that time that Jatin’s clarion call “Amra morbo, jagat jagbe” (We shall die to awaken the nation) evoked the rising currents of Indian nationalism. Thousands of young revolutionaries joined Jatin’s brand of the freedom movement.

Jugantar, which soon became a pan-India movement, galvanized the spirit of strident

nationalism. Jugantar Party successfully set up its units across India and even spread far across southeast Asia, Europe, and America. The mounting serial attacks on British Raj by Jugantar shook the colonial administration right to London.

Jatin was arrested in the Alipore bomb case but released soon. Then he was arrested again in

connection with the Howrah conspiracy case and locked in Howrah jail. While in Howrah jail, he came in contact with fellow revolutionaries belonging to various groups who were operating in different parts of Bengal. After spending 11 months in jail, Jatin was acquitted and released from jail in 1911.

o The Howrah-Sibpur Conspiracy case refers to the arrest and trials of 47 Bengali Indian

nationalists of the Anushilan Samiti that followed in the wake of the murder of Inspector Shamsul Alam on 24 January 1910 in Calcutta.

Jatin was in touch with Indian revolutionaries like Rash Behari Bose, Lala Har Dayal, MN Roy,

Chempakaraman Pillai, Nair San, and Shyamji Krishna Varma, who was working for Indian independence from overseas.

In 1912, Jatin met the German Crown Prince in Kolkata and asked him for arms for an insurrection, in order to create a socialist government in India. The German Crown Prince promised to provide arms and funds for the proposed Indian insurrection against the British.

o The task of obtaining funds and armaments was entrusted to MN Roy, the key lieutenant of Jatin. In April 1915, Roy left India in search of German armaments which were believed to be en route,

somewhere in the Pacific. The plan was indeed fantastic. Bagha Jatin had plans to sever communications between Madras and Bengal.

Hence option (a) is the correct answer.

25.  Consider the following pairs:

Tribal Revolt Associated        Leaders

1. Santhal Rebellion :             Sidhu and Kanhu Murmu

2. Ramosi Uprisings :              Thamman-Dora

3. Munda Uprisings :              Birsa Munda

4. Koya Revolts :                      Chittur Singh

How many pairs given above are correctly matched?

(a) Only one pair

(b) Only two pairs

(c) Only three pairs

(d) All four pairs

Answer: B

The Santhal, who lived in the area between Bhagalpur and Rajmahal hills, known as ‘Daman-i-koh’, rose in revolt; made a determined attempt to expel the outsiders the dikus, and proclaimed the complete ‘annihilation’ of the alien. In 1855, the Santhals gathered in the Bhagnadihi Village under the leadership of the brothers – Sidhu and Kanhu Murmu – and declared themselves free from the colonial rule. At the outset, the British rule was paralyzed in the area and the native agents were killed. The rebellion covering the districts of Birbhum, Singbhum, Bankura, Hazaribagh, Bhagalpur, and Monghyr in Orissa and Bihar was precipitated mainly by economic causes. This great insurrection is known as the ‘Hul’. Hence pair 1 is correctly matched.

The Ramosi’s, the hill tribes of the Western ghats had not reconciled to the British Rule and the British pattern of administration. They rose under Chittur Singh in 1822 and plundered the country around Satara. Again, there were eruptions in 1825-26 and the disturbances continued till 1829. Hence pair 2 is not correctly matched.

Munda Rebellion is one of the prominent 19th-century tribal rebellions in the subcontinent. Birsa Munda led this movement in the Chota Nagpur Plateau region south of Ranchi from 1899-1900. The ulgulan, meaning ‘Great Tumult’, sought to establish Munda Raj and independence. The Mundas traditionally enjoyed a preferential rent rate as the khuntkattidar or the original clearer of the forest. But in course of the 19th century, they had seen this khuntkatti land system being eroded by the jagirdars and thikadars coming as merchants and moneylenders. Hence pair 3 is correctly matched.

o The government attempted to redress the grievances of the Mundas through the survey and settlement operations of 1902-10. The Chhotanagpur Tenancy Act of 1908 provided some recognition to their khuntkatti rights and banned beth begari. Chhotanagpur tribals won a degree of legal protection for

their land rights.

The Koyas of the eastern Godavari track (now Andhra) revolted frequently against the

administration aided by Khonda Sara leaders. The revolt started against the ‘Muttadars’ (zamindars) who formed a chain of rent collectors from the colonial rulers in the year 1862.

o The Britishers deprived the tribals of their traditional rights over the toddy trees -the most valuable

property of the tribals for they yielded drinks. The traders from the region took advantage of the

situation, by extending loans to the tribals they confiscated their produce and cattle.

o As a result, the tribals attacked the authorities under the leadership of Thamman-Dora in 1879. Their grievances included police and moneylender persecution, new restrictions, and rejection of their

traditional rights to forest regions. Hence pair 4 is not correctly matched.

26.  Who among the following established the Widow Remarriage Association, in the 1850s?

(a) Vishnushastri Pandit

(b) Debendranath Tagore

(c) Karsondas Mulji

(d) Ishwar Chandra Vidyasagar

Answer: A

Vishnushastri Pandit founded the Widow Remarriage Association in the 1850s in Bombay.

o The main task of this association was to make a concerted effort to eradicate people’s apathy,

cowardice, and bigotry standing in the way of the remarriage movement. Hence, option (a) is the correct answer.

Another prominent worker in this field was Karsondas Mulji who started the Satya Prakash in Gujarati in 1852 to advocate widow remarriage.

Pandit Ishwar Chandra Vidyasagar (1820-91), the principal of Sanskrit College, Calcutta, advocated for widow remarriage and due to his efforts that the Hindu Widows’ Remarriage Act, 1856, was passed. It legalized the marriage of widows and declared issues from such marriages as legitimate. Vidyasagar cited Vedic texts to prove that the Hindu religion sanctioned widow remarriage.

27.  With reference to different perspectives about the formation of the Indian National Congress (INC) in 1885, consider the following statements:

1. According to the Safety Valve theory, the INC was formed by A.O. Hume for releasing the growing discontent of the Indians.

2. Lala Lajpat Rai opposed Safety Valve theory and believed that the INC was formed by Indians to promote national interest.

Which of the statements given above is/are correct?

(a) 1 only

(b) 2 only

(c) Both 1 and 2

(d) Neither 1 nor 2

Answer: A

The Safety Valve theory advocates that A. O. Hume a retired British official formed the Congress with the idea that it would prove to be a ‘safety valve’ for releasing the growing discontent of the Indians. To this end, he convinced Lord Dufferin not to obstruct the formation of the Congress. Hence statement 1 is correct.

Many leaders including Lala Lajpat Rai believed in the Safety Valve theory. Lala Lajpat Rai maintained that the Indian National Congress was organised to serve as a ‘safety valve’ for the growing unrest in the country and strengthen the British Empire. According to him the redress of political grievances and political advance of India was only a by-product and of secondary importance. Hence statement 2 is not correct.

 

28.  Which of the following is correct regarding the Age of Consent Act, 1891?

(a) The Act forbade the marriage of girls below the age of 12.

(b) The Act was passed during the time of Lord Ripon.

(c) The Act was passed due to the efforts of Bal Gangadhar Tilak.

(d) B.M. Malbari opposed the passage of the bill.

Answer: A

The Age of Consent Act, 1891, also known as Act X of 1891, was legislation enacted in British India in 1891 which raised the age of consent for sexual intercourse for all girls, married or unmarried, from ten to twelve years in all jurisdictions, its violation subject to criminal prosecution as rape. Hence, option (a) is the correct answer.

While an 1887 case in a Bombay high court of a child bride Rukhmabai renewed discussion of such a law it was the death of an eleven-year-old Bengali girl Phulmoni Dasi in 1889 that drove intervention by the British. The act was passed in 1891.

Behramji M. Malabari (1853- 1912) spoke vigorously against child marriage and widow remarriage among Hindus. It was his efforts that led to the Age of Consent Act regulating the age of consent for females.

Bal Gangadhar Tilak opposed the Age of Consent Bill. His objection was mainly that such reforms must come from people governing themselves and not under an alien rule.

The Age of Consent Act was signed in 1891 during the viceroyship of Lord Lansdowne.

29.  With respect to the Indian states, arrange the following in chronological order of acceptance of subsidiary alliance system:

1. Sindhia

2. Holkar

3. Mysore

4. Awadh

Select the correct answer using the code given below.

(a) 4-3-1-2

(b) 4-3-2-1

(c) 3-4-2-1

(d) 3-4-1-2

Answer: D

The Indian princes who accepted the subsidiary system were: the Nizam of Hyderabad (September 1798 and 1800), the ruler of Mysore (1799), the ruler of Tanjore (October 1799), the Nawab of Awadh (November 1801), the Peshwa (December 1801), the Bhonsle Raja of Berar (December 1803), the Sindhia (February 1804), the Rajput states of Jodhpur, Jaipur, Macheri, Bundi and the ruler of Bharatpur (1818). The Holkars were the last Maratha confederation to accept the Subsidiary Alliance in 1818. Hence option (d) is the correct answer.

 

30.  Consider the following pairs:

Women’s Organisation          Associated Person

1. Indian Women’s Association   : Annie Besant

2. Bharat Stree Mandal                 : Sarala Devi Choudharani

3. Arya Mahila Samaj                     : Mehribai Tata

How many pairs given above are correctly matched?

(a) Only one pair

(b) Only two pairs

(c) All three pairs

(d) None of the pairs

Answer: B

Pair 1 is correctly matched: The Women’s Indian Association (WIA) was founded at Adayar,

Madras, in 1917 by Annie Besant, Margaret Cousins, Jeena Raja Dasa, and others to liberate women from the deplorable condition women suffered in socio-economic and political matters during the 19th and the early 20th century. . The WIA was one of the first organizations to boldly connect Indian women’s social and sexual subjugation with patriarchy, poverty, and political disenfranchisement.

Pair 2 is correctly matched: In 1910, Sarla Devi Chaudhurani convened the first meeting of the Bharat Stree Mahamandal in Allahabad. Considered as the first major Indian women’s organisation set up by a woman, its objectives included the promotion of education for women, abolition of the purdah system and improvement in the socio-economic and political status of women all over India. Sarla Devi believed that the man working for women’s upliftment lived ‘under the shade of Manu’.

Pair 3 is not correctly matched: Pandita Ramabai Saraswati (1858-1922), founded the Arya Mahila Samaj to serve the cause of women. She pleaded for improvement in the educational syllabus of Indian women before the English Education Commission which was referred to Queen Victoria. This resulted in medical education for women which started in Lady Dufferin College. Later Ramabai Ranade established a branch of Arya Mahila Samaj in Bombay.

o Ramabai Ranade (1863-1924) founded the Ladies Social Conference (Bharat Mahila Parishad), under the parent organisation National Social Conference, in 1904 in Bombay. She married Justice Mahadev Govind Ranade and at the age of 18, she joined the Prarthana Samaj founded by her husband a liberal hub for 19th-century Maharashtra. In 1909, Ramabai Ranade founded Seva Sadan Society.

 

31.  In the context of the proposal of introducing the ryotwari system in the south and

southwestern India, what were the reasons behind implementing the ryotwari system

over the zamindari system in these areas?

1. In these regions there were no zamindars present with a large estate with whom land settlement could be made.

2. Under the permanent settlement the company was the ultimate financial loser as it had to share revenue with zamindars.

3. It was a continuation of the state of affairs related to land revenue that had existed in the past for these areas.

Select the correct answer using code given below.

(a)    1 and 2 only

(b)   2 and 3 only

(c)    1 and 3 only

(d)   1, 2 and 3

 

Answer: D

 

The establishment of British rule in South and South Western India brought new problems of land

settlement. While framing land revenue policies for these areas British officials believed that in these regions, there were no Zamindars with large estates with whom settlement of land revenue could be made and the introduction of the zamindari system would upset the existing state of affairs. Hence statement 1 is correct.

Many Madras officials led by Reed and Thomas Munro recommended that settlement should therefore be made directly with actual cultivators.

They also pointed out that under the Permanent settlement the Company was a financial loser as it had to share the revenues with the zamindars and could not claim a share of the growing income from land. Moreover, the cultivator was left at the mercy of the zamindar who could oppress him at will. Hence statement 2 is correct.

Munro claimed that it is the system that has always prevailed in India. The supporters of the Ryotwari system also claimed that it was a continuation of the state of affairs that had existed in the past. Due to these factors, Munro introduced the Ryotwari system in the parts of Bombay and Madras presidency in the beginning of the 19th Century. Hence statement 3 is correct.

 

32.  Which of the following statements is/are correct regarding Third Anglo Maratha war?

1. The chief reason for this war was theBritish conflict with the Pindaris.

2. After the war, Maratha confederacy was dissolved and the peshwaship was abolished.

Select the correct answer using the code given below:

(a) 1 only

(b) 2 only

(c) Both 1 and 2

(d) Neither 1 nor 2

 

Answer: C

 

After the second Anglo-Maratha war, the Marathas made one last attempt to rebuild their old prestige.

The Third Anglo-Maratha War (18171819) was the final and decisive conflict between the English East India Company and the Maratha Empire in India.

Marathas wanted to retake all their old possessions from the English. They were also unhappy with the British resident’s interference in their internal matters. The chief reason for this war was the British conflict with the Pindaris whom the British suspected were being protected by the Marathas. Hence statement 1 is correct.

The Peshwa was defeated at Khirki, Bhonsle at Sitabuldi, and Holkar at Mahidpur. Some important treaties were signed. These were:

o June 1817, Treaty of Poona, with Peshwa.

o November 1817, Treaty of Gwalior, with Sindhia.

o January 1818, Treaty of Mandsaur, with Holkar.

In June 1818, the Peshwa finally surrendered and the Maratha confederacy was dissolved. The

peshwaship was abolished. Peshwa Bajirao became a British retainer at Bithur, near Kanpur. Hence

statement 2 is correct.

The territories annexed from the Pindaris became the Central Provinces under British India.

This war led to the end of the Maratha Empire. All the Maratha powers surrendered to the British.This was one of the last major wars fought and won by the British. With this, the British controlled most parts of India barring Punjab and Sindh directly or indirectly.

 

33.  The Indian Councils Act of 1861 provided for

1. Inclusion of non-official members for legislative purpose

2. Voting powers to the Central Legislative Council in relation to a few aspects of the budget

3. Introduction of the portfolio system

Which of the statements given above are correct?

(a) 1 and 2 only

(b) 1 and 3 only

(c) 2 and 3 only

(d) 1, 2 and 3

 

 

Answer: B

 

Indian Councils Act, 1861 provided the appointment of additional members for legislative purposes. The viceroy could add six to twelve additional members, of whom at least half had to be non-officials who could be either Indian or English. Hence statement 1 is correct.

However, the legislative council so constituted possessed no real powers and was merely advisory in nature. It could not discuss important matters and no financial matters at all without the previous approval of the Government. It had no control over the budget. Thus, it had no voting powers over the budget. Hence statement 2 is not correct.

Before the adoption of the portfolio system in the Government of India, all Governmental business was disposed of by the Governor-General in Council. As the amount and complexity of the business of the Government increased, the work of the various Departments was distributed amongst the members of the Council (Portfolio system). This procedure was legalized by the Indian Councils Act, 1861 during the time of Lord Canning, leading to the introduction of the portfolio system and the inception of the Executive Council of the Governor-General. Hence statement 3 is correct.

The Morley Minto Reforms of 1909 & not the Indian Councils Act of 1861, for the first time provided for the appointment of one Indian to the viceroy’s executive council (Satyendra Sinha was the first Indian to be appointed in 1909). It provided for the appointment of an Indian member to the Governor General Executive Council.

 

 

 

 

 

 

34.  Consider the following statements regarding the Aravipuram Movement:

1. The movement was a reaction against Brahmanic supremacy and the caste system.

2. It was started by E.V. Ramaswamy Naicker.

Which of the statements given above is/are correct?

(a) 1 only

(b) 2 only

(c) Both 1 and 2

(d) Neither 1 nor 2

 

 

Answer: A

 

Aravipuram Movement was launched by Sri Narayana Guru on Shivaratri day of 1888. It was a

reaction against Brahmanic supremacy and the caste system. Hence, statement 1 is correct and

statement 2 is not correct.

Sri Narayana Guru defied the religious restrictions traditionally placed on the Ezhava community and consecrated an idol of Shiva at Aravipuram.

The Aravipuram Pratistha was a historic event, because a member of a lower caste, forbidden from entering the temple, had himself consecrated the Shiva image in a temple.

Millions saw Sri Narayana Guru as a saint, seer, philosopher, poet and social reformer. Education and organisation were amongst his many slogans for freedom and strength. He held that the essence of all religions is one and the same, and advocated the comparative study of all faiths.

The organisation which he set up to maintain and manage the institutions he had founded later became known as Sri Narayana Dharma Paripalana Yogam (Society for the Propagation of Sri Narayana Guru’s Tenets).

 

35.  Which one of the following statements is not correct regarding the foreign policy of British India?

 

(a) Lord Auckland advocated for a ‘forward policy’ while dealing with Afghanistan.

(b) John Lawrence started a policy of ‘masterly inactivity’ as a reaction to the disasters of the First Afghan War.

(c) Lord Ripon followed the foreign policy of ‘proud reserve’ aimed at safeguarding the spheres of influence.

(d) Lord Curzon followed a ‘realistic and common sense’ policy towards the tribesmen of the North-West Frontier.

 

 

Answer: C

 

Lord Auckland who came to India as the governor-general in 1836, advocated a forward policy.

o This implied that the Company government in India itself had to take initiatives to protect the boundary of British India from a probable Russian attack.

o This objective was to be achieved either through treaties with the neighbouring countries or by annexing them completely. This prompted the British government to go ahead with the forward policy, and Tripartite Treaty (1838) was entered into by the British,Sikhs and Shah Shuja (who had been deposed from the Afghan throne in 1809 and had been living since then as a British pensioner at Ludhiana).

John Lawrence (1864-1869) started a policy of masterly inactivity which was a reaction to the disasters of the First Afghan War and an outcome of practical common sense and intimate knowledge of the frontier problem and of Afghanpassion for independence.

Lord Lytton started a new foreign policy of ‘proud reserve’, which was aimed at having scientific frontiers and safeguarding‘spheres of influence’. According to Lytton, the relations with Afghanistan could no longer be left ambiguous. Hence, option (c) is the correct answer.

Lytton made an offer of a favourable treaty to Sher Ali, but the Amir wanted friendship with both his powerful neighbours, Russia and British India, while keeping both of them at arm’s length. Later, Sher Ali refused to keep a British envoy in Kabul while having earlier granted a similar concession to the Russians.

o Lytton was displeased, and when the Russians withdrew their envoy from Kabul, Lytton decided to

invade Afghanistan.

o Sher Ali fled in face of the British invasion, and the Treaty of Gandamak (May 1879) was signed with Yakub Khan, the eldest son of Sher Ali.

o Treaty of Gandamak (May 1879): The treaty signed after the Second-Anglo Afghan War

provided that:

the Amir conduct his foreign policy with the advice of the Government of India;

a permanent British resident be stationed at Kabul; and

the Government of India gives Amir all support against foreign aggression and an annual subsidy.

Under Lord Curzon, the frontier policy was revised and a new system came into operation. He followed a realistic and common sense’ policy towards the tribesmen. He would have nothing to do with the ‘elastic and pliable adjectives’ of Masterly Inactivity or Forward Policy or surrender to the “paralysing influence of these labels. He said: “Let our new frontier policy be called by any name that men choose. Only let it be based not upon obsolete political formulas, but upon up-to-date common sense.”

o In the light of past experience, Curzon hoped “to draft a code of frontier policy which could with

consistency, and without violent interruptions, be applied to the whole line of North-Western frontier

from the Pamirs to Baluchistan.” Curzon’s policy may be summed up as a policy of ‘military

concentration as against diffusion, and of tribal conciliation in place of exasperation’.

 

36.  Consider the following statements regarding the Sanyasi rebellion:

 

1. The immediate cause of the revolt was the restrictions imposed by the British upon pilgrims visiting holy places.

2. Anandamath, a novel by Bankim Chandra Chattopadhyay, is based on this rebellion.

3. Lord Warren Hastings was the Governor-General of Bengal during this rebellion.

Which of the statements given above is/are correct?

 

(a) 1 and 2 only

(b) 1 and 3 only

(c) 2 only

(d) 1, 2 and 3

 

 

Answer: D

 

The Sanyasis rose in rebellion after the great famine of 1770 in Bengal which caused acute chaos and

misery. The Bengal famine of 1770 led peasants whose lands were confiscated, displaced zamindars, and disbanded soldiers, and poor to come together in a rebellion. They were joined by the Sanyasis (who were originally peasants) and Fakirs.

However, the immediate cause of the rebellion was the restrictions imposed by the British upon pilgrims visiting holy places among both Hindus and Muslims. Hence statement 1 is correct.

Two famous Hindu leaders who supported them were Bhawani Pathak and a woman, Devi

Choudhurani. They attacked English factories and seized their goods, cash, arms and ammunition.

Maznoom Shah was one of their prominent leaders.

o It was only after a prolonged action that Lord Warren Hastings (Governor-General of Bengal) could subdue the sanyasis. Equal participation of Hindus and Muslims characterized the uprisings. Hence statement 3 is correct.

Anandamath, a semi-historical novel by Bankim Chandra Chattopadhyay, is based on the Sanyasi Revolt. Bankim Chandra also wrote a novel, Devi Chaudhurani, as he saw the importance of women too taking up the struggle against an alien rule that posed a threat to traditional Indian values. Hence statement 2 is correct.

 

37.  With reference to changes introduced into the tribal economy by the British Administration in India, which of the following reasons compelled tribals to rise against the British rulers?

1. The British administration restricted the entry of a large number of moneylenders, traders, and revenue farmers as middlemen in the tribal areas.

2. The Indian Forest Act of 1865 extended British colonialism’s claim over forests in India.

3. The British administration introduced a system of forced labour (begar) in tribal areas.

 

Which of the statements given above is/are correct?

(a) 2 only

(b) 2 and 3 only

(c) 3 only

(d) 1, 2 and 3

 

Answer: B

 

The cause of the tribal violence was directed towards the money-lenders and traders who were seen as

extensions of the colonial government. As agriculture was extended in a settled form by the Company government, the tribals lost their land, and there was an influx of non-tribals to these areas due to colonial revenue policies. The money lenders introduced in the tribal areas led to severe exploitation of the local tribals. They became bonded labourers under the new economic system. The tribal societies had a system of joint ownership of land which was replaced by the notion of private property. Hence statement 1 is not correct.

Another common cause was the resentment against the imposition of laws by the ‘foreign government’ that was seen as an effort at destroying the tribal’s traditional socio-economic framework. The Indian Forest Act of 1865 extended British colonialism’s claim over forests in India.

o Its main aim was an assertion of the monopoly of the British over the Indian forest land, and it gave the government undisputable power to regulate the forest and pastures. It enabled the

Revenue and Forest Department to control the entire forest and grazing land. Some forests were

classified as Reserved Forests for they produced timber that the British wanted. In these forests, people were not allowed to move freely, practice jhum cultivation, collect fruits, or hunt animals. Hence statement 2 is correct.

The Begar system of forced labour was introduced in tribal areas as well that had altered labour relations. An influx of Christian Missionaries was supported by the British. The reason behind intense tribal rebellion is that zamindars, the Police, the revenue, and the court alas have exercised a combined system of extortions, oppressive exactions, forcible dispossession of property, abuse, and personal violence, and a variety of petty tyrannies upon the tribals. Hence statement 3 is correct.

Under British rule, the functions and powers of the tribal chiefs changed considerably. They were allowed to keep their land titles over a cluster of villages and rent out lands as zamindars but they lost much of their administrative power and were forced to follow laws made by British officials in India.

o They also had to pay tribute to the British, and discipline the tribal groups on behalf of the British. They lost the authority they had earlier enjoyed amongst their people and were unable to fulfil their traditional functions.

All these reasons compelled tribals to rise against the British rulers.

 

 

 

 

38.  Which of the following statements is/are correct regarding the Pitt’s India Act, 1784?

1. The Board of Control was established.

2. The orders of the Board of Control became binding on the Court of Directors.

3. The number of members in the Governor General’s Council was reduced.

Select the correct answer using the code given below.

(a) 1 only

(b) 2 and 3 only

(c) 1 and 2 only

(d) 1, 2 and 3

 

 

Answer: D

 

The Pitt’s India Act, 1784 also called the East India Company Act, 1784 was passed by the British

Parliament to correct the defects of the Regulating Act of 1773.

Provisions:

o This act resulted in dual control of British possessions in India by the British government and the

Company with the final authority resting with the government.

o A Board of Control consisting of six members was constituted to take care of civil and military

affairs, and would include one of the Secretary of State (Board President), the chancellor of the

exchequer and four privy councillors. Hence statement 1 is correct.

o The Governor-General’s council’s strength was reduced to three from four members. One of the three would be the Commander-in-Chief of the British Crown’s army in India. Hence statement 3 is correct.

o The orders of the board became binding on the Court of Directors, which was required to send all its letters and dispatches to the board for its perusal. Hence statement 2 is correct.

 

39.  Consider the following statements:

1. The first railway line in British India was laid from Bombay to Thane in 1853.

2. The railways in India were built using British capital with a promise of a guaranteed minimum return of 5% on their paid-up capital.

Which of the statements given above is/are correct?

(a) 1 only

(b) 2 only

(c) Both 1 and 2

(d) Neither 1 nor 2

 

 

Answer: C

 

The phase of economic exploitation of India between the closing decades of the nineteenth century till 1947 has been termed the Period of Finance Capital. During this phase, the Capital inflow became another instrument of Indian exploitation.

There was no investment of British capital in India till 1833 i.e., throughout the period the Company

traded. The flow of British capital into India became significant only after 1857 and penetrated into diverse economic fields in the latter half of the 19th century but tightened its tentacles over India in the 20th century in the form of finance capital. Thus, there was a capital flow though not significant prior to the revolt of 1857. For example, the first railway line was laid from Bombay to Thane in 1853 with the help of the British capital. Hence statement 1 is correct.

British capital investment in India normally took two basic forms:

o Loans were raised in England by the Secretary of State on behalf of the Indian Government and by semi-public organisations mostly for investment in railways, irrigation, developments of ports, and

hydro-electric projects.

o Foreign business investments in India.

In 1849, the colonial state of India entered an agreement with the Great Indian Peninsular Railway Company and the East India Railway Company to construct the railway from Bombay to Thane and from Howrah to Raniganj coal fields. These experimental agreements were then extended to other companies to develop railways in other parts of the country. They were given a state guarantee of a minimum return of 5% on their paid-up capital. Hence statement 2 is correct.

 

40.  Consider the following statements regarding Naujawan Bharat Sabha:

1. Bhagat Singh was the founding secretary of the Sabha.

2. It actively collaborated with Kirti Kisan Party to organise workers and peasants.

Which of the statements given above is/are correct?

(a) 1 only

(b) 2 only

(c) Both 1 and 2

(d) Neither 1 nor 2

 

 

Answer: C

 

The Naujawan Bharat Sabha was formed to channelize the militant nationalist movement on ideological lines in March 1926 by Bhagat Singh. In this task, he was ably assisted by Bhagwati Charan Vohra, Dhanwantri, Ehsan Elahi and others. Ram Krishna and Bhagat Singh became its first president and secretary respectively. Hence statement 1 is correct.

The Sabha had two-fold objectives-social and political. The social objectives comprised the

popularization of swadeshi goods, plain living, physical fitness, inculcation of the sense of brotherhood and the stimulation of interest in Indian languages and civilization.

The political programme of the Sabha included the following:

o to establish a completely independent republic of the labourers and peasants of the whole of India

o to infuse a spirit of patriotism into the hearts of the youths of the country in order to establish a united Indian nation.

o to organise the labourers and peasants.

Later Naujawan Bharat Sabha actively collaborated with Kirti Kisan Party for the cause of workers and peasants from 1928 onwards to infuse a spirit of struggle and to make the peasantry and workers conscious of their rights, the Sabha took a very keen interest in organizing workers’ and peasants’ conferences on various issues. The bond between the Kirti party and the Naujawan Bharat Sabha was further strengthened when Sohan Singh Josh, leader of the Kirti party, was elected chairman of the Amritsar branch of the Sabha in July, 1928. Hence statement 2 is correct.

 

41.  In the context of revolutionary activities during freedom struggle, consider the following statements regarding the Gopinatha Saha case of 1924:

1. Gopinath Saha was arrested for writing seditious article encouraging violence.

2. At Mahatma Gandhi’s instance, All India Congress Committee passed a resolution criticising his action.

3. Gopinath Saha was acquitted of all charges after the trial.

Which of the statements given above is/are correct?

(a) 2 only

(b) 1 and 2 only

(c) 1 and 3 only

(d) 1, 2 and 3

 

 

Answer: A

 

Among the several ‘actions’ of the reorganized groups was the attempt to assassinate Charles Tegart, the hated Police Commissioner of Calcutta, by Gopinath Saha in January 1924. By an error, another Englishman named Day was killed. Hence statement 1 is not correct.

The Government came down on the people with a heavy hand. A large number of people, suspected of being terrorists, or their supporters, were arrested under a newly promulgated ordinance. These included Subhash Chandra Bose and many other Congressmen. Saha was hanged despite massive popular protest. The revolutionary activity suffered a severe setback. Hence statement 3 is not correct.

The Gopinath Saha Case almost resulted in a second split in the Indian National Congress. The principle of non-violence was clearly violated when the Bengal unit of the Congress, prompted by Chittaranjan Das, praised what it called Saha’s self-sacrifice, setting, as it were, the cat among the pigeons.

A resolution was moved at the All India Congress Committee at Gandhi’s instance in June 1924, which characterized Saha’s action as misguided love of the country and disapproved emphatically of all political murders as inconsistent with the Congress creed. This resolution could be passed by 78 in favour and 70 against. Though Gandhi won, he considered the passing of the resolution by a razor thin margin as his defeat. Hence statement 2 is correct.

 

42.  Consider the following statements about the Communist Party of India (CPI):

1. It was founded in Kanpur in1925.

2. Its activities were supported by the Britishers.

Which of the statements given above is/are correct?

(a) 1 only

(b) 2 only

(c) Both 1 and 2

(d) Neither 1 nor 2

 

 

Answer: A

 

The CPI was formally founded in 1925 at the Indian Communist Conference in Kanpur. Hence,

statement 1 is correct.

o The government crackdown on communists resulted in the arrest and trial of 31 leading communists, trade unionists, and left-wing leaders in 1929; they were tried in Meerut in the famous Meerut conspiracy case.

o Workers’ and peasants’ parties sprouted up across the country, spreading Marxist and communist ideas.

o All of these communist groups and workers’ and peasants’ parties remained a part of the national

movement and collaborated with Congress.

o The British government did not favor the activities of the Communists in India and on 23rd July 1934, it imposed a ban on the functioning of the party. Hence, statement 2 is not correct.

 

43.  Consider the following pairs:

Works                                               Authored by

1. Bandi Jiwan                          : Sachindra Nath Sanyal

2. Philosophy of Bomb           : Batukeshwar Dutt

3. Pather Dabi                          : Sarath Chandra Chattopadhyay

Which of the pairs given above are correctly matched?

(a) 1 and 2 only

(b) 1 and 3 only

(c) 2 and 3 only

(d) 1, 2 and 3

 

 

Answer: B

 

Sachindra Nath Sanyal was the founder of the Hindustan Republican Association. He was sent to the dreaded Cellular Jail in the Andamans and in jail he wrote the famous book “Bandi Jiwan” (A Life of Captivity). This book would become the bible for a generation of revolutionaries fighting British rule. Hence pair 1 is correctly matched.

The Philosophy of Bomb was written by Bhagwati Charan Vohra in response to Mahatma Gandhi’s article the Cult of Bomb. Hence pair 2 is not correctly matched.

Pather Dabi is a Bengali novel written by Sarath Chandra Chattopadhyay published between 1922- 26. The book is about a secret society named Pather Dabi whose goal is to free India from British rule. Hence pair 3 is correctly matched.

 

44.  He led the Mahad Satyagraha to challenge the regressive customs of the caste Hindus. He also established the Bahishkrit Hitakarini Sabha in 1924 to highlight the difficulties and grievances of the Dalits before the government. Its motto was: ‘Educate, Agitate and Organise’.

Who among the following is being described in the above passage?

(a) Jyotiba Phule

(b) B R Ambedkar

(c) Mahadeo Govind Ranade

(d) Mahatma Gandhi

 

 

Answer: B

 

Babasaheb Ambedkar, who had experienced the worst form of casteist discrimination during his childhood, fought against upper caste tyranny throughout his life. He organised the All India Scheduled Castes Federation, while several other leaders of the depressed classes founded the All India Depressed Classes Association. Ambedkar condemned the hierarchical and insular caste system as a whole, and advocated the annihilation of the institution of caste for the real progress of the nation. The struggle of the depressed classes led to the provision of special representation for these classes in the Government of India Act, 1935.

Dr Bhimrao Ambedkar led the Mahad Satyagraha in March 1927 to challenge the regressive customs of the caste Hindus. He stressed the necessity of removing ideas of ‘high’ and ‘low’ and inculcating selfelevation through self-help, self-respect and self-knowledge. He led a procession of some 2,500 ‘untouchables’ through the town of Mahad to the Chawdar tank, a public source of water tank from which the untouchables were not allowed to draw water. Dr Ambedkar took water from the tank and drank it.

There were huge protests by caste Hindus. Later in December 1927, Ambedkar and his colleagues burnt the ‘Manusmriti’ at the same place as a gesture of getting rid of inequalities.

Dr Ambedkar established the Bahishkrit Hitakarini Sabha in 1924 to highlight the difficulties and grievances of the dalits before the government. Its motto was: ‘Educate, Agitate and Organise’. Hence, option (b) is the correct answer.

 

45.  With reference to the Capitalist class’s role during the freedom struggle, consider the following statements:

1. During the Swadeshi movement capitalists were opposed to the boycott agitation.

2. Capitalist class largely supported the Civil Disobedience Movement of the 1930s.

3. Capitalist class preferred a constitutional mode of struggle.

4. In 1928, the capitalists refused to support the Government in introducing the Public Safety Bill.

Which of the statements given above are correct?

(a) 1 and 2 only

(b) 2 and 3 only

(c) 1 and 3 only

(d) 1, 2, 3 and 4

 

 

Answer: D

 

The Indian capitalists’ attitude had undergone significant changes on this issue over time. During the

Swadeshi Movement (1905-08), the capitalists remained opposed to the boycott agitation. Even during the Non-Cooperation Movement of the early 1920s, a small section of the capitalists, including

Purshottamdas, openly declared themselves enemies of the Non- Cooperation Movement. However, during the 1930s Civil Disobedience Movement, the capitalists largely supported the movement and refused to respond to the Viceroy’s exhortations to publicly repudiate the Congress stand and his offer of full guarantee of government protection against any harassment for doing so. Hence statements 1 and 2 are correct.

Indian capitalist class had its own notions of how the anti-imperialist struggle ought to be waged. It was always in favour of not completely abandoning the constitutional path and the negotiating table and generally preferred to put its weight behind constitutional forms of struggle as opposed to mass civil disobedience. This was due to several reasons. Hence statement 3 is correct.

o There was the fear that mass civil disobedience, especially if it was prolonged, would unleash forces which could turn the movement revolutionary in a social sense.

o Capitalists were unwilling to support a prolonged all-out hostility to the government of the day as it

prevented the continuing of day-to-day business and threatened the very existence of the class.

The increasing radicalization of the Congress in the Left direction in the 1930s, with the growing influence of Nehru, and the Socialists and Communists within the Congress, spurred the capitalists into becoming more active in the political field.

o The fear of radicalization of the national movement, however, did not push the capitalists into the ‘lap of imperialism,’ as predicted by contemporary radicals and as actually happened in some other colonial and semi-colonial countries. Instead, the Indian capitalists evolved a subtle, many-sided strategy to contain the Left, no part of which involved a sell-out to imperialism or imperial interests.

o For example, in 1928, the capitalists refused to support the Government in introducing the Public Safety Bill, which was intended to contain the Communists, on the ground that such a provision would be used to attack the national movement. Hence statement 4 is correct.

 

46.  Consider the following statements regarding the Indian National Congress (INC):

1. The Surat Session, 1907 was presided over by Rash Behari Ghosh.

2. Alfred Webb was the first non-Indian President of the Indian National Congress.

Which of the statements given above is/are correct?

(a) 1 only

(b) 2 only

(c) Both 1 and 2

(d) Neither 1 nor 2

 

Answer: A

The foundation of the Indian National Congress in 1885 was not a sudden event or a historical accident. It was the culmination of a process of political awakening that had its beginning within the 1860s and 1870s and took a serious breakthrough within the late 1870s and early 1880s.

o It was founded in December 1885 and was the first organized expression of the Indian National

Movement on an all-India scale.

In the Surat session, conducted at the banks of river Tapi, Extremists demanded that Lala Lajpat Rai be elected as the president of the Congress. But the Moderates were successful in getting elected Rash Behari Ghosh as the president. Hence statement 1 is correct.

George Yule was the first non-Indian President of the Indian National Congress. He presided over the Allahabad session of 1888. Alfred Webb presided over the Madras session of 1894. Hence statement 2 is not correct.

 

 

 

 

 

 

 

 

47.  With reference to the Doctrine of Lapse, consider the following statements:

1. The sanction of the British must be obtained by the adopted son to inherit the personal property of the chieftain.

2. The adopted son would be entitled to same pension that his father had been receiving.

3. The Doctrine was introduced for the first time by Lord Dalhousie.

Which of the statements given above is/are correct?

(a) 2 and 3 only

(b) 3 only

(c) 1 only

(d) None

 

 

Answer: D

The Doctrine of Lapse was an annexation policy followed widely by Lord Dalhousie when he was

India’s Governor-General from 1848 to 1856. It was used as an administrative policy for the extension of British Paramountcy.

Features of Doctrine of Lapse:

o According to this doctrine, any princely state under the direct or indirect (as a vassal) control of the East India Company, should the ruler not produce a legal male heir, would be annexed by the company.

o As per this, any adopted son of the Indian ruler could not be proclaimed as heir to the kingdom.

The adopted son would only inherit his foster father’s personal property and estates. Dalhousie

recognised the right of the adopted son to succeed to the personal property of the Chieftan and for this

sanction was not required. Hence statement 1 is not correct.

o The adopted son would also not be entitled to any pension that his father had been receiving or to any of his father’s titles. Hence statement 2 is not correct.

o This was not introduced by Lord Dalhousie even though it was he who documented it and used it

widely to acquire territories for the British. As early as 1834 the Court of Directors had laid down that in case of failure of lineal successors the permission ‘to adopt’ was an indulgence that “should be the exception, not the rule, and should not be granted but as a special mark of favour and approbation”. Few years later in 1841, the Home authorities decided in favour of a uniform policy. It was in pursuance of  the policy thus laid down that Mandavi state was annexed in 1839, Kolaba and Jalaun in 1840 and the titular dignity of the Nawab of Surat abolished in 1842. Hence statement 3 is not correct.

Dalhousie’s contribution was that he uniformly applied this Doctrine and did not ignore or neglect any opportunity in consolidating the territories of the East India Company.

 

48.  In the context of India’s socio-religious reforms history, Paramahansa Mandali aimed at

(a) breaking caste rules.

(b) promoting the approach of Vedic culture, Upanishads, and the predominance of Aryan society.

(c) propagating the ideas of Ramakrishna Paramahansa.

(d) preservation of the status quo in socio religious aspect

 

 

Answer: A

 

Paramahansa Mandali Founded in 1849 in Maharashtra, the founders of the Paramahansa

MandaliDadoba Pandurang, Mehtaji Durgaram and othersbegan as a secret society that worked to reform Hindu religion and society in general. The founders of the mandali were primarily interested in breaking caste rules. The ideology of the society was closely linked to that of the Manav Dharma Sabha. Besides believing that one god should be worshipped, society also said real religion is based on love and moral conduct. Hence option (a) is the correct answer.

At their meetings, food cooked by lower caste people was taken by the members. This mandalis also

advocated widow remarriage and women’s education. Branches of Paramahansa Mandali existed in Poona, Satara, and other towns of Maharashtra.

Mahatma Jyotiba Phule founded the Satyashodhak Samaj (Truth Seekers’ Society) in 1873, with the leadership of the samaj coming from the backward classes, malis, telis, kunbis, saris and dhangars. This movement gave a sense of identity to the depressed communities as a class against those upper-caste people who used religion and the blind faith of the masses to exploit the masses for personal monetary gain. Phule aimed at the complete abolition of the caste system and socio-economic inequalities; he was against Sanskritic Hinduism. The samaj rejected the approach of Vedic culture, Upanishads, and the predominance of Aryan society.

Narayan Malhar Joshi founded the Social Service League in Bombay with an aim to secure for the masses better and reasonable conditions of life and work. Joshi also founded the All India Trade Union Congress (1920) to organize workers against British policies.

Dharma Sabha was formed in 1830 in Calcutta by Radhakanta Deb. The organization was established mainly to counter the ongoing social reform movements led by protagonists such as Raja Ram Mohan Roy and Henry Derozio. The focus of the association was on the preservation of the status quo in socioreligious matters of Hindu Society against intrusion by the British administration.

 

49.  Consider the following statements:

1. The English got trading privileges in this region by a Mughal farman of 1630.

2. Apprehensive of a Franco-Russian joint land invasion of India, the British signed a “Treaty of Eternal Friendship” in 1807 with this region, to create a buffer.

3. By the 1840s, this region was brought under the English control by use of force.

Which of the following regions is being referred to in the statements given above?

(a) Rohilkhand

(b) Afghanistan

(c) Balochistan

(d) Sindh

 

 

Answer: D

 

In the early 19th century, the English started to show an interest in Sindh where they enjoyed some trade facilities authorised by a farman of the Mughal Emperor in 1630. The farman provided the English with such privileges in the ports of Sindh which they enjoyed elsewhere. This advantage was enjoyed by the English upto 1775 when a not-too-friendly ruler, Sarfraz Khan, made the English close their factory.

In June 1807, the alliance of Tilsit with Alexander I of Russia was joined by Napoleon Bonaparte. The alliance had as one of its conditions a combined invasion of India by the land route. Now the British wanted to create a barrier between Russia and British India. To achieve this, Lord Minto sent three delegations under the leadership of various prominent persons to forge alliances. Accordingly, Metcalfe was sent to Lahore, Elphinstone to Kabul and Malcolm to Tehran. Sindh was visited by Nicholas Smith who met the Amirs to conclude a defensive arrangement. After negotiations, the Amirs agreed to a treatytheir first ever treaty with the English.

In 1843, under Governor-General Ellenborough, Sindh was merged into the British Empire and Charles Napier was appointed its first governor.

Hence option (d) is the correct answer.

 

50.  Who among the following were the members of the South Indian Liberal Federation?

1. P. Ananda Charlu

2. P. Tyagaraja Chetti

3. M. Veeraraghavachari

4. C.N. Mudaliar

Select the correct answer using the code given below.

(a) 1 and 2 only

(b) 2 and 3 only

(c) 1 and 4 only

(d) 2 and 4 only

 

 

Answer: D

 

Justice Party, officially the South Indian Liberal Federation, was a political party in the Madras

Presidency of British India. The South Indian Liberal Federation (SILF) was started by T. M. Nair, P. Thyagaraya Chetti, and C. Natesa Mudaliar in 1916 against the domination of brahmins in government service, education and political field. Hence, option (d) is the correct answer.

It opposed the Congress as a Brahman-dominated organisation and claimed separate communal

representation for the non-Brahmans as had been granted to the Muslims in the Morley-Minto reform. This demand, supported by the colonial bureaucracy, was granted in the Montagu-Chelmsford reform of 1919, as it allocated twenty-eight reserved seats to the non-Brahmans in the Madras Legislative Council.

The party also played a vital role in allowing women to contest elections paving the way for Dr.

Muthulakshmi Reddy to become the first woman legislator in India.

o The pioneering efforts of Dr. Muthulakshmi Reddy saw the abolition of Devadasi system when theJustice party was in power.

In 1938, E. V. Ramaswamy ( Periyar) was elected the leader of the Justice party. In 1944, Justice Party along with the Self Respect movement was renamed Dravidar Kazhagam.

 

51.  With reference to the Ilbert Bill during the British colonial rule in India, which one of the following statements is correct?

(a) The bill was introduced during the tenure of Lord Lytton.

(b) C.P. Ilbert was the Secretary of British India at the time of introduction of the bill.

(c) It proposed to give Indian session judges the power to try European offenders in small towns.

(d) The bill was passed by the Indian Legislative Council in its original form without any amendments.

 

 

Answer: C

 

The Ilbert Bill was proposed in 1883 by the British Government headed by Lord Ripon. C.P Ilbert, the law member in his council, introduced in February 1883 what is known as the infamous Ilbert Bill. The bill sought to abolish “judicial disqualification based on race distinctions”. It proposed to give Indiandistrict magistrates and session judges the power to try European offenders in the mofussil (small towns), as they already did in the presidency towns. Hence option (c) is the correct answer.

However, the bill was met with opposition from the European community. In January 1884 Ripon ultimately succumbed to the pressure and withdrew the bill, substituting it with a milder compromise formula, which somehow sought to preserve the principle by adding a provision of trial by a mixed jury in such cases involving European offenders. It became clear to the nationalists that justice and fair play could not be expected where the interests of the European community were involved. The Ilbert Bill controversy proved an eye-opener to the Indian intelligentsia. It became clear to them that justice and fair play could not be expected where the interests of the European community were involved.

 

 

 

 

 

52.  In the context of Anglo Mysore rivalry, consider the following statements:

1. Treaty of Mangalore was signed between Haider Ali and the British after 2nd Anglo Mysore war.

2. Haider Ali took the help of the French to set up an arms factory at Dindigul.

3. Under the treaty of Seringapatam, Nizam got the regions surrounding the Tungabhadra and its tributaries.

Which of the statements given above is/are correct?

(a) 1 and 2 only

(b) 2 only

(c) 1 and 3 only

(d) None

 

 

Answer: B

 

Second Anglo Mysore War (1780-84)

o Background: Haidar Ali accused the English of breach of faith and non- observance of the Treaty of Madras when in 1771 he was attacked by the Marathas, and the English failed to come to his

aid. Haidar Ali took the help of the French to set up an arms factory at Dindigul (now in Tamil

Nadu), and also introduced Western methods of training for his army. Haidar Ali’s friendship with the French caused even more concern to the English. They therefore tried to capture Mahe, which Haidar regarded to be under his protection. Haidar considered the English attempt to capture Mahe a direct challenge to his authority. Hence statement 2 is correct.

o Haidar forged an anti-English alliance with the Marathas and the Nizam. He followed it up by an attack in the Carnatic, capturing Arcot, and defeating the English army under Colonel Baillie in 1781.

o Haider Ali died of cancer on December 7, 1782. His son, Tipu Sultan, carried on the war for one year without any positive outcome. Fed up with an inconclusive war, both English and Tipu Sultan opted for peace, negotiating the Treaty of Mangalore (March, 1784) under which each party gave back the territories it had taken from the other. Hence statement 1 is not correct.

Third Anglo Mysore War (1790-92)

o With the support of the Marathas and the Nizam, the English attacked Seringapatam. Tipu offered

serious opposition, but the odds were against him. Consequently, he had to pay heavily under the Treaty of Seringapatam.

o The war ended after the 1792 Siege of Seringapatam and the signing of the Treaty of

Seringapatam, under this treaty, nearly half of the Mysorean territory was taken over by the victors.

Baramahal, Dindigul and Malabar went to the English, while the Marathas got the regions

surrounding the Tungabhadra and its tributaries and the Nizam acquired the areas from the

Krishna to beyond the Pennar. Hence statement 3 is not correct.

 

53.  With reference to trading stations/factories established by various European powers, consider the following pairs:

Location of factory                  Associated European power

1. Shrirampur            :                             Danish

2. Chinsura                 :                             Dutch

3. Chandranagore     :                             French

4. Chittagong             :                             Portuguese

Which of the pairs given above is/are correctly matched?

(a) 1 and 2 only

(b) 3 only

(c) 1, 2 and 4 only

(d) 1, 2, 3 and 4

 

 

Answer: D

 

 

The Danish East India Company

o The Danes entered as traders in 1616 but with no ambition to establish an Empire. They managed to secure the Tranquebar port from the Nayak of Tanjore in 1620 and built a fort there. They also

established their factories at Masulipatam, Porto Novo and Shrirampur(1755). However, their success was limited, and ultimately they sold off their factories to the English and finally quit India in

1845. Hence pair 1 is correctly matched.

The Dutch East India Company

o The Dutch East India Company was formed in 1602 through a charter. The Dutch were primarily

interested in spice trade. Therefore, they paid more attention to the Far East. India was just a trading

depot for them. They established their factories at Pulicat (1610), Cambay (1620), Surat and Agra

(1621), Hariharpur (1633), Patna (1638). Dacca (1650), Udaiganj (1651), Chinsura (1653),

Qasimbazar, Baranagore, Balasore and Negapatam (1659-60). Hence pair 2 is correctly matched.

The French East India Company

o The French were late comers to the Eastern trade. The French East India Company was founded in

1664. The first French factory was establish at Surat in 1668. This was the place of prime importance

to the English. In 1669, the French established their second factory at Masulipatam. In 1673, they got

Pondicherry, and in 1674 the Nawab of Bengal granted them a site near Calcutta where in 1690-92 they built the town of Chandranagore. Hence pair 3 is correctly matched.

The Portuguese East India Company

o The Portuguese were the very first European naval adventurers on the scene, who had started out on daring sea voyages down the coast of Africa back from 1415 AD. In 1528, the Sultan of Bengal

permitted the Portuguese to establish factories and customs houses in the Port of Chittagong. A fort

and naval base was established in Firingi Bandar. The settlement grew into the most prominent Eurasian port on the Bay of Bengal during the Age of Discovery. Hence pair 4 is correctly matched.

 

54.  With reference to the Charter Act of 1793, consider the following statements:

1. Under the Act, a sum of one lakh rupees was to be set aside for the promotion of learning among Indians.

2. The act provided for the creation of Maal Adalats or revenue courts.

3. The act provided for the abolition of slavery in India.

Which of the statements given above is/are correct?

(a) 1 only

(b) 2 and 3 only

(c) 1 and 3 only

(d) None

 

 

Answer: D

 

Provisions of the Charter Act of 1793

o The British Charter Act of 1793 was the attempt by the British government to regulate the affairs of the East India Company, which had acquired great power and influence in the Indian subcontinent since its first voyage in the early 17th century. It was the first step in the process of nationalisation of the British East India Company.

o The Act renewed the Company’s commercial privileges for next 20 years.

o The Company, after paying the necessary expenses, interest, dividends, salaries, etc., from the Indian revenues, was to pay 5 lakh pounds annually to the British government.

o The royal approval was mandated for the appointment of the governor-general, the governors, and the commanderin-chief.

o Senior officials of the Company were debarred from leaving India without permissiondoing so was treated as resignation.

o The Company was empowered to give licenses to individuals as well as the Company’s employees to trade in India. The licenses, known as ‘privilege’ or ‘country trade’, paved the way for shipments of opium to China.

o It established the Company’s trade monopoly with India. And expanded the scope of the Supreme

Court’s jurisdiction in India.

o The revenue administration was separated from judicial functions, resulting in the abolition of the Maal Adalats, or revenue courts. Hence statement 2 is not correct.

o The Governor-General was given more powers. He could override his council’s decision under

certain circumstances. He was also given authority over the governors of Madras and Bombay. When

the Governor-General was present in Madras or Bombay, he would supersede in authority over the

governors of Madras and Bombay.

A sum of one lakh rupees was to be set aside for the revival, promotion and encouragement of

literature, learning and science among the natives of India, every year under the Charter Act of

1813. Hence statement 1 is not correct.

Slavery had been abolished in Britain in 1820 and the Slavery Abolition Act 1833 provided for the gradual abolition of slavery in most parts of the British Empire. In India, the Indian Slavery Act of 1843 abolished slavery. The Act proclaimed slavery to be illegal and made indulgence in slavery a penal offence. Hence statement 3 is not correct.

However, after the abolition of slavery in 1843, a system of indentured labour was used to recruit people as plantation labour in other British colonies. Under this system, labourers were hired on contract for a period of five years (indenture) and they could return to their homeland at the end of the period.

 

55.  Consider the following statements regarding the Trade Disputes Act (TDA), 1929:

1. The Act provided for a system of tribunals and a ban on strikes.

2. Bhagat Singh threw a bomb in the Central Legislative Assembly against the passage of this enactment.

Which of the statements given above is/are correct?

(a) 1 only

(b) 2 only

(c) Both 1 and 2

(d) Neither 1 nor 2

 

 

Answer: C

 

Alarmed at the increasing strength of the trade union movement under the extremist influence, the

government resorted to legislative restrictions. It passed the Public Safety Ordinance (1929) and the Trade Disputes Act (TDA), 1929.

The TDA, 1929 provided for

o the compulsory appointment of Courts of Inquiry and Consultation Boards for settling industrial

disputes

o and made illegal the strikes in public utility services like posts, railways, water and electricity unless each individual worker planning to go on strike gave advance notice of one month to the administration;

o forbade trade union activity of coercive or purely political nature and even sympathetic strikes. Hence, statement 1 is correct.

Bhagat Singh and B.K. Dutt were asked to throw a bomb in the Central Legislative Assembly on 8 April 1929 against the passage of the Public Safety Bill and the Trade Disputes Bill which would reduce the civil liberties of citizens in general and workers in particular. Hence, statement 2 is correct.

 

56.  Consider the following statements with respect to the Battle of Adyar:

1. It was part of the third Carnatic war fought between the British and the French.

2. It led to the French occupation of St. George fort of Madras.

Which of the statements given above is/are correct?

(a) 1 only

(b) 2 only

(c) Both 1 and 2

(d) Neither 1 nor 2

 

Answer: D

 

The Battle of Adyar took place on in October 1746. The battle was between the French East India

Company men and Nawab of Arcot forces over the St. George Fort, which was held by the French. It was part of the First Carnatic War between the English and the French. Hence statement 1 is not correct.

o The First Carnatic War (1740-48) was an extension of the Anglo-French War in Europe which was

caused by the Austrian War of Succession.

The English navy under Commodore Bennett seized some French ships to provoke France. France

retaliated by seizing Madras in 1746 with the help of the fleet from the French colony of Mauritius.

The Battle was launched by the Nawab because he was furious that the French Governor, Joseph Dupleix, had attacked Madras without his permission and captured St. George Fort. Nawab of Arcot, a close ally of the British, set out to regain it by sending troops, led by his son Mahfuz Khan, to Madras.

French troops, with disciplined firing and then charging with bayonets defeated Mahfuz Khan’s troops. So, the Battle of the Adyar River, which began on the morning of October 24, 1746, ended by that evening,

with the French occupation of Fort St. George consolidated. Hence statement 2 is not correct.

Battle of Adyar proved a turning point in the Indian history because for the first time, techniques of 18th century European warfare, developed in Prussia and tested on the battlefields of France and Flanders, had been tried out in India.

o Nothing in the Mughal armoury could match the techniques of 18th century European warfare,

particularly the invention of screws for elevating the guns gave the artillery greater precision and

increased the fire power of the foot soldier, giving them an edge in the battle against the cavalry.

o 700 French sepoys defeated the Mughal Army of 10,000 troopers, which was seen never before in India.

 

57.  With reference to a leader of the Indian National Movement, consider the following statements:

1. He was chosen as the first Secretary of the Native Press Association formed in the 1870s.

2. He became a member of the Imperial Legislative Assembly in 1921.

3. He cleared the Imperial Civil Service (ICS) but was not allowed to work in the administration.

Who among the following has been described in the statements given above?

(a) Ananda Mohan Bose

(b) Pheroze Shah Mehta

(c) Surendranath Banerjea

(d) Sisir Kumar Ghosh

 

Answer: C

 

Surendranath Banerjea (1848-1925) was born on 10 November 1848 in Calcutta. He graduated from Calcutta University in 1868 and proceeded to England to compete for the Indian Civil Services. He cleared the Imperial Civil Service (ICS) but was not allowed to work in the administration. He was declared disqualified by the British.

He was the first to familiarize an Indian audience with Giuseppe Mazzini (1805-1872), the pioneer of Italian unification. Banerjea was the first leader to use material from Indian and foreign histories to instil a sense of patriotism in his audience. On his return to India in June 1875, Banerjea began his new career as a Professor of English. In 1876 he founded the Indian Association. He was among the earliest leaders who attempted to forge all India links among the political associations in different parts of India.

The Indian Association of Calcutta tried to open branches in the other two presidencies with a view to sending a joint petition to the British parliament on the eve of the renewal of the Company’s Charter.

The Native Press Association was founded by the Indian journalists in the 1870s and Surendranath Banerjea was chosen as its first secretary. The Indian Association organised a National conference in Calcutta in 1883 and another was scheduled in December 1885. He became a member of the Imperial Legislative Assembly in 1921 and was knighted the same year.

Hence option (c) is the correct answer.

 

58.  With reference to the Treaty of Amritsar (1809), consider the following statements:

1. It was signed between Maharaja Ranjit Singh of Punjab and the British East India Company.

2. The treaty demarcated the river Sutlej as the boundary between the British and the state of Punjab.

3. The state of Punjab was forced to accept a permanent British resident at the royal court of Punjab.

Which of the statements given above is/are correct?

(a) 1 and 2 only

(b) 2 only

(c) 3 only

(d) 2 and 3 only

 

Answer: A

 

The Treaty of Amritsar (1809) was signed on April 25, 1809, between Maharaja Ranjit Singh ( (1780 1839) and the British East India Company. It was a pact between Charles T. Metcalfe and Maharaja Ranjit Singh. Maharaja Ranjit Singh was the founder of the Sikh Empire. He was the son of Mahan Singh, the leader of the Sukarchakiya misl. Ranjit Singh brought under control the area extending from the Sutlej to the Jhelum. He conquered Lahore in 1799 and Amritsar in 1802. Ranjit Singh proved to be an efficient administrator. He greatly modernised his army with the help of Europeans. Hence statement 1 is correct.

Ranjit Singh died in 1839. His successors could not keep the state intact and, soon enough, the British took control over it. Maharaja Daleep Singh was the son and successor of Maharaja Ranjit Singh who came to the throne in 1843.

As per the provisions of the Treaty of Amritsar (1809), the river Sutlej was the boundary between the British and the state of Punjab. The treaty settled Indo-Sikh relations for a generation. The immediate occasion was the French threat to northwestern India, following Napoleon’s Treaty of Tilsit with Russia (1807) and Ranjit’s attempt to bring the area around Sutlej states under his control. Hence statement 2 is correct.

The treaty of Amritsar did not have any provision regarding the British resident at the royal court of Punjab. The treaty of Lahore (1846) signed after the first Anglo- Sikh War (1845-46), provided for the stationing of a British resident at Lahore. The state of Punjab was annexed by Lord Dalhousie in 1849 after the Second Anglo- Sikh War (1848-49) and the eleven-year-old Maharaja, Daleep Singh was pensioned off to England. Hence statement 3 is not correct.

 

59.  The Madras Native Association was set up during the nineteenth century in Madras as a branch of which of the following associations?

(a) The Bombay Association

(b) British Indian Association of Calcutta

(c) Indian League

(d) Poona Sarvajanik Sabha

Answer: B

 

The Madras Native Association was established in 1852 as a platform for educated Indians to protest against any injustice on the part of the British. It was the first Indian political organization in the Madras

Presidency.

It was set up as a branch of the British Indian Association of Calcutta. The Madras Association also sent a petition to the Parliament on the eve of the passing of the Charter Act of 1853 making demands similar to that of the British Indian Association and the Bombay Association right from its inception was worked by some officials, possessed very little vitality, had hardly any hold upon the public mind and languished into obscurity after 1857.

The British Indian Association of Calcutta was formed in 1851 through the merger of the Landholders’ Society and the Bengal British India Society.

Other important political association in Madras was the Madras Mahajan Sabha founded in 1884 by M. Viraraghavachari, B. Subramaniya Aiyer and P. Anandacharlu.

Hence option (b) is the correct answer.

 

60.  Consider the following pairs:

Organisation Associated                                                     member

1. The Bombay Presidency Association           : Badruddin Tyabji

2. Madras Mahajan Sabha                                  : G. Subramaniya Aiyer

3. The Bangabhasha Prakasika Sabha              : Raja Ram Mohan Roy

Which of the pairs given above are correctly matched?

(a) 1 and 2 only

(b) 2 and 3 only

(c) 1 and 3 only

(d) 1, 2 and 3

 

Answer: A

 

The Bombay Presidency Association was started by Badruddin Tyabji, Pherozshah Mehta and K.T. Telang in 1885. It was founded in response to Lytton’s reactionary policies and the Ilbert Bill

controversy. The association has always had cordial relations with the Poona Sarvajanik Sabha. The

Bombay Presidency Association, the Poona Sarvajanik Sabha, the Madras Mahajana Sabha, and the Indian Association of Calcutta sent a joint deputation to England in September 1885 to present India’s case to the British electorate. It championed Indian interests and also hosted the first meeting of the Indian National Congress in Bombay at the end of 1885. Hence pair 1 is correctly matched.

The Madras Mahajan Sabha was founded in 1884 by M. Viraraghavachari, G. Subramaniya Aiyer and P. Anandacharlu. The Mahajana Sabha held its first conference between 29 December 1884 and 2 January 1885. It demanded fundamental rights of Indians such as national freedom and other common social issues for the welfare of our fellowmen since 1884. In September 1885, the Sabha in collaboration with the Bombay Presidency Association and the Indian Association sent a delegation to England. On April 22, 1930, the Sabha organized the Salt Satyagraha movement in Madras‘ George Town, Esplanade, High Court, and Beach areas. Hence, pair 2 is correctly matched.

The Bangabhasha Prakasika Sabha was formed in 1836 by associates of Raja Rammohan Roy. Raja Ram Mohan Roy died in 1833. Prasanna Kr. Thakur, Kalinath Chowdhury, Dwarakanath

Tagore, and others were the associates of Raja Rammohan Roy who founded the Bangabhasa Prakasika Sabha in 1836. The Bangabhasha Prakasika Sabha is regarded as the earliest political organisation in India.

It sought to increase the association of Indians in the administration, the spread of education, pushing

forward Indian demands to the British parliament. Hence pair 3 is not correctly matched

                                                                                                                                          

61.  Which one of the following statements is not correct with reference to the early demands of the Moderates?

(a) They demanded a reduction in land revenue and salt duty

(b) They demanded policies to help in the growth of Indian industries and handicrafts

(c) They demanded the repeal of the Arms Act

(d) They demanded to abolish Permanent Settlement from all parts of India

 

 

Answer: D

 

The Congress programme during the first phase (1885-1905) was very modest. It demanded moderate constitutional reforms, economic relief, administrative reorganization, and defense of civil rights. Some of the important demands were as follows,

·        Organization of the provincial councils.

·        Simultaneous examination for the I.C.S. in India and England.

·        Reduction in land revenue and salt duty.

·        Abolition or reconstitution of the Indian Council.

·        Separation of the Judiciary from the executive.

·        Repeal of the Arms Act.

·        Appointment of Indians to the commissioned ranks in the Army.

·        Reduction of military expenditure and

·        Introduction of Permanent Settlement to other parts of India.

But with respect to the Permanent Settlement, the moderates did not demand the abolishment of Permanent Settlement from all parts but rather demanded the extension to other parts. So, Option (d) is not correct.

 

62.  Consider the following statements with reference to the Rowlatt Act Satyagraha:

1. It was completely a non-violent satyagraha throughout India.

2. The Satyagraha Sabha concentrated mainly on publishing propaganda literature and collecting signatures on the Satyagraha pledge.

3. In Ahmedabad, the government enforced martial law.

Which of the statements given above is/are correct?

(a) 1 and 3 only

(b) 2 and 3 only

(c) 2 only

(d) 1, 2 and 3

 

 

Answer: B

 

Satyagraha was to be launched on April 6, 1919, but before it could be launched, there were large-scale violent, anti-British demonstrations in Calcutta, Bombay, Delhi, Ahmedabad, etc. Especially in Punjab, the situation became so very explosive due to wartime repression, forcible recruitments, and ravages of disease that the Army had to be called in. April 1919 saw the biggest and the most violent anti-British upsurge since 1857. So, Statement 1 is not correct.

The entire agitation against the Rowlatt act proved its unorganized nature whereby the Satyagraha Sabha of Mahatma Gandhi concentrated mainly on publishing propaganda literature and collecting signatures on the Satyagraha pledge. So, Statement 2 is correct.

Mahatma Gandhi left Bombay on April 8 to promote the Satyagraha agitation in Delhi and Punjab. But, as his entry into Punjab was considered dangerous by the government, Gandhi was removed from the train he was traveling at Palwal near Delhi and taken back to Bombay. The news of Gandhi’s arrest precipitated the crisis. The situation became tense in Bombay, and violence broke out in Ahmedabad and Virangam. In Ahmedabad, the government enforced martial law. In particular, the Punjab region and Amritsar witnessed the worst scenes of violence. So, Statement 3 is correct.

 

63.  Consider the following statements with reference to the Swaraj Party of 1923 :

1. Chittaranjan Das was the Secretary of the party.

2. It was described as a rival organization to the Indian National Congress.

3. It was successful in bringing an amendment to the Act of 1919.

Which of the statements given above is/are not correct ?

(a) 1 only

(b) 2 and 3 only

(c) 1 and 2 only

(d) 1, 2 and 3

 

 

Answer: D

 

The Swaraj Party was a political party formed in India in January 1923 following the Indian National Congress’s annual Congress in Gaya in December 1922. It was formed by Chittaranjan Das and Motilal Nehru. Chittaranjan Das was its President, and Motilal Nehru was its secretary. Both leaders decided to contest legislative council seats in elections. Their goal was to disrupt a foreign administration. So, Statement 1 is not correct.

The Swarajists were allowed to contest elections as a group within Congress. The Swarajists accepted the Congress program with only one difference – that they would join legislative councils. But the party was described as a party within the Congress and not as a rival organization. So, Statement 2 is not correct.

The Swaraj Party’s objective was to enter the Council by participating in the general elections at the end of 1923 to block the Government of India Act of 1919 from within the Council. But they neither amend nor end the Government of India Act 1919. So, Statement 3 is not correct.

 

64.  Consider the following statements with reference to the Nehru Report of 1928 :

1. It provided for the separation of Sind from Bombay without any conditions

2. It provided for a federal form of Government in India with Residuary powers to be vested in State

3. Under the report, the senate will comprise two hundred members elected for seven years

Which of the statements given above is/are correct ?

(a) 1 and 3 only

(b) 3 only

(c) 1 and 2 only

(d) 2 only

 

 

Answer: B

 

A committee under the leadership of Motilal Nehru was formed to outline the principles on which the constitution was to be drafted. The draft constitution prepared by the committee is known as the Nehru Report. One of its recommendations is that Sind should be separated from Bombay but with certain conditions like it should be financially self-sufficient to get separated. So, Statement 1 is not correct.

The report stated that there should be a Federal form of Government in India with Residuary powers to be vested in the Centre. There will be no separate electorate for minorities because it awakens communal sentiments, therefore, it should be scrapped and a joint electorate should be introduced. Therefore, the federal form of Government in India with Residuary powers is not to be vested in State. So, Statement 2 is not correct.

The Nehru Report of 1928 mainly recommended that the senate will comprise two hundred members elected for seven years, while the House of Representatives should consist of five hundred members elected for five years. Governor-General will act on the advice of the executive council. It was to be collectively responsible to the parliament. So, Statement 3 is correct.

 

65.  With reference to the Home Rule Movement of 1916, Consider the following statements

1. Home Rule League of Bal Gangadhar Tilak was active in Maharashtra and Karnataka

2. The movement never stated the goal of complete independence

3. It concentrated on oppressing colonial policies

Which of the statements given above is/are correct ?

(a) 2 and 3 only

(b) 1 and 2 only

(c) 1, 2 and 3

(d) 3 only

 

 

Answer: C

 

In April 1916, Bal Gangadhar Tilak set up his Indian Home Rule League. In Belgaum, Tilak conducted his first Home Rule meeting. His league’s headquarters were in Poona. His league was restricted to Maharashtra (except Bombay), Karnataka, the Central Provinces, and Berar. So, Statement 1 is correct.

The Home Rule movement never explicitly stated its goal is to get complete independence.

The major goals and objectives of the Home Rule Movement are as follows,

·        To promote political education in India.

·        To achieve self-government in India.

·        To discuss agitation for self-government.

·        To bring a revolution among Indians and raise their voice against the British Government’s suppression.

·        To maintain the principles of the Indian National Congress and revive Indian political activity.

·        To urge the British government for greater political representation of Indians.

So, Statement 2 is correct.

The Home Rule League was popular with moderates because it promoted political debate and education. The league also advocated for separate electorates and minority representation in legislatures, to which the Congress agreed. The Home Rule League was active all year, as opposed to the Congress Party, which was only active once a year.

The movement brought political awareness to more areas of the country. This movement led to the Montagu Declaration of 1917, in which it was declared that there would be more Indians in the government leading to the development of self-governing institutions, ultimately realizing responsible governments in India. The Home Rule League focuses on the oppression of colonial policy through opposition to government policy, such as forest regulations and liquor laws. So, Statement 3 is correct.

 

66.  Consider the following statements:

1. The policies of the British Indian Association, formed in 1851, were against landlords in Bengal.

2. Indian National Association, founded by Surendranath Banerjee, demanded the simultaneous holding of civil service examinations in England and India.

Which of the statements given above is/are correct ?

(a) 1 only

(b) 2 only

(c) Both 1 and 2

(d) Neither 1 nor 2

 

Answer: B

 

·        British Indian Association was founded on October 29, 1851, at Calcutta with Raja Radha Kanta dev and Debendranath Tagore as its President and Secretary, respectively. In 1851, the Landholders’ Society and the Bengal British India Society merged into the British Indian Association. The object of the Association was ‘to secure improvements in the local administration of the country and the system of government laid down by Parliament. But also, this association predominantly supports the landlords and the upper class and they were obviously concerned with those measures of the administration that affected their class interests. So, Statement 1 is not correct.

·        The Indian Association of Calcutta (also known as the Indian National Association) superseded the Indian League. It was founded in 1876 by younger nationalists of Bengal led by Surendranath Banerjea and Ananda Mohan Bose, who were getting discontented with the conservative and pro-landlord policies of the British Indian Association.

·        The Indian National Association aimed to promote by every legitimate means the people’s political, intellectual, and material advancement. The Indian National Association’s contributions are:

o   It protested against the reduction of the age limit in 1877 for candidates of the Indian Civil Service examination.

o   The association demanded simultaneous holding of civil service examinations in England and India.

o   Indianisation of higher administrative posts.

o   It led a campaign against the repressive arms act and the vernacular press act.

So, Statement 2 is correct.

 

67.  Consider the following statements:

1. Simla Deputation, led by Agha Khan, demanded excess representation in the legislative council in response to their high representation in defense.

2. G.K.Gokhale met John Morley and demanded a self-governing system similar to other British colonies.

3. The Indian Councils Act of 1909 increased the number of directly elected members in the Central and Provincial legislature.

Which of the statements given above is/are correct?

(a) 1 and 2 only

(b) 2 and 3 only

(c) 1 and 3 only

(d) 1, 2 and 3

 

 

Answer: A

 

·        In October 1906, a group of Indian Muslim leaders organized to meet Lord Minto called the Simla Deputation, led by the Agha Khan. They demanded separate electorates for the Muslims and representation above their numerical strength because of the value of the contribution Muslims made to the defence of the empire. The same group quickly took over the Muslim League, initially floated by Nawab Salimullah of Dacca along with Nawabs Mohsin-ul-Mulk and Waqar-ul-Mulk in December 1906. So, Statement 1 is correct.

·        Gopal Krishna Gokhale went to England to meet the Secretary of State for India, John Morley, to put Congress’s demands of a self-governing system and emphasize the need for reforms similar to that in the other British colonies. So, Statement 2 is correct.

·        One of the reforms of the Indian Councils Act of 1909 was that the number of indirectly elected members in the central Legislative Council and the Provincial Legislative Councils increased. In the provincial councils, a non-official majority was introduced, but since some of these non-officials were nominated and not elected, the non-elected majority remained. So, Statement 3 is not correct.

 

68.  Consider the following statements:

1. All India trade union congress (AITUC) was founded in October 1920.

2. Lala Lajpat Rai was elected as the first president of AITUC.

3. In the 1937 elections, the AITUC had opposed the congress candidates.

Which of the statements given above is/are correct?

(a) 1 and 2 only

(b) 2 and 3 only

(c) 1 and 3 only

(d) 1, 2 and 3

 

Answer: A

 

·        The All-India Trade Union Congress was founded on October 31, 1920. The Indian National Congress president for the year, Lala Lajpat Rai, was elected as the first president of AITUC and Dewan Chaman Lal as the first general secretary. So, Statements 1 and 2 are correct.

·        After 1931 there was a dip in the working-class movement because of a split in 1931 in which the corporatist trend led by N.M. Joshi broke away from the AITUC to set up the All-India Trade Union Federation.

·        During the 1937 elections, the AITUC supported the Congress candidates. The Congress ministries were generally sympathetic to the workers’ demands. Much legislation which was favorable to the workers was passed. So, Statement 3 is not correct.

 

69.  Which one of the following trials/cases weakened the working class movement ?

(a) Meerut conspiracy case of 1929

(b) Banaras conspiracy case of 1915

(c) Dacca conspiracy case of 1910

(d) Howrah conspiracy case of 1908

 

 

Answer: A

 

The Meerut Conspiracy Case was a court case that was initiated in British Raj in March 1929 and decided in 1933. In March 1929, the Government arrested 31 labour leaders, and the three-and-a-half-year trial resulted in the conviction of Muzaffar Ahmed, S.A. Dange, Joglekar, Philip Spratt, Ben Bradley, Shaukat Usmani, and others. This trial is known as Meerut Conspiracy Case. The trial got worldwide publicity but weakened the working-class movement. So, Option (a) is correct.

 

70.  Consider the following statements with reference to the Lucknow Pact of 1916:

1. Congress accepted separate electorates for Muslims in the legislature.

2. Congress accepted the system of weightage and reservation of seats for minorities in the legislature.

Which of the statements given above is/are correct?

(a) 1 only

(b) 2 only

(c) Both 1 and 2

(d) Neither 1 nor 2

 

 

Answer: C

·        In the Lucknow Session of the Indian National Congress, which happened in 1916, the Muslim League and the Congress were come together and presented common demands to the government. This was known as the Lucknow Pact of 1916.

·        The League agreed to present joint constitutional demands with the Congress to the government, and the Congress accepted the Muslim League’s position on separate electorates, which would continue till any one community demanded joint electorates.

·        The Muslims were also granted a fixed proportion of seats in the legislatures at all-India and provincial levels. In this pact, Congress accepted separate electorates and the system of weightage and reservation of seats for minorities in the legislature. Congress accepted the principle of separate electorates for Muslims. So, Statements 1 and 2 are correct.

 

71.  Consider the following statements with respect to Chittagong Armoury Raid :

1. The raid was led by Surya Sen along with Anant Singh, Ganesh Gosh, Pritilata Waddedar and others.

2. It was undertaken in the name of the Hindustan Socialist Republican Association.

3. It lacked the support of Muslims

4. Kalpana Dutt associated with the raid, was given life imprisonment.

Which of the statements given above are correct?

(a) 1 and 2 only

(b) 2 and 3 only

(c) 1, 3 and 4 only

(d) 1 and 4 only

 

 

Answer: D

 

·        Surya Sen planned to organize an armed rebellion to demonstrate that British rule could be challenged through arms. So Surya Sen led the raid with revolutionary youths, including Anant Singh, Ganesh Ghosh, Ambika Chakravarty and Loknath Paul.

·        Their action was carefully prepared and included the occupation of the two main armories in Chittagong and arming a large band of revolutionaries with the seized arms. So, Statement 1 is correct.

·        The raid was taken in the name of the Indian Republican Army, Chittagong Branch. All the revolutionaries gathered outside the Police Armouries. Surya Sen was formally declared the President of the Provincial Revolutionary Government. So, Statement 2 is not correct.

·        The Chittagong group included many Muslims, such as Sattar, Mir Ahmed, Fakir Ahmed Mian, and Tunu Mian. Surya Sen and his comrades were given active and massive support by Muslim villagers, enabling them to avoid and resist arrest for nearly three years. So, Statement 3 is not correct.

·        The new phase of revolutionary terrorism in Bengal made advancements in three aspects. There was large-scale participation of young women.Women’s acted as the messenger and carried guns and also fought with a gun in their hands Pritilata Waddedar died while conducting a raid on the Railway Institute at Paharatali, while Kalpana Datta was arrested and tried along with Surya Sen and given life imprisonment. So, Statement 4 is correct.

 

72.  Consider the following Pairs:

List I                                                    List II

1. Surendranath Banerjee     – Nation in Making

2. M.K. Gandhi                         – Hind Swaraj

3. Lala Lajpat Rai                    – The Indian Struggle

How many pairs given above is/are correctly matched ?

(a) Only one pair

(b) Only two pairs

(c) All three pairs

(d) None of the pairs

 

 

Answer: B

 

·        Nation in Making book was written by Surendarnath Banerjee. Another book written by him includes The Trumpet Voice of India. So, Pair (1) is correct.

·        Hind Swaraj (1909) is the only book Gandhi wrote in Gujarati and translated himself. Other books by him include My Experiments with Truth. So, Pair (2) is correct.

·        Netaji Subash Chandra Bose wrote the Indian Struggle. An Indian Pilgrim is another book authored by Subash Chandra bose. So, Pair (3) is not correct.

 

73.  The objective of the Muddiman Committee of 1924 was to:

(a) Impose censorship on national press

(b) Define the powers of the Secretary of State for India

(c) Investigate the working of dyarchy

(d) Improve the relationship between the Government of India and the Indian States

 

 

Answer: C

 

·        The Government of India appointed Muddiman Committee at the beginning of 1924. It was established to investigate the working of the Constitution as set up in 1921 under the India Act of 1919.

·        The committee was set up due to the growing political unrest on the dyarchy issue of the Constitution. The committee was also known as the Reforms Enquiry Committee. It had Sir Alexander Muddiman as the chairman, who served as the then Home Member of the Government of India. Muddiman Committee presented its report in December 1924.

·        In the report, most members suggested only minor changes in the structure of the Constitution. On the other hand, the minority, consisting entirely of non-official Indians, condemned the dyarchy and advocated for its immediate abolition. The members also laid down the democratization of the Constitution.

·        After the committee started its operation, a resolution was pressed in the Imperial legislature for the revision of the Constitution to secure India’s full self-governing Dominion status. So, Option (c) is correct.

 

74.  Consider the following statements:

1. C.R.Das was the political guru for Netaji Subash Chandra Bose.

2. Gopala Krishna Gokhale was the Political guru for M.G. Ranade.

Which of the statements given above is/are incorrect?

(a) 1 only

(b) 2 only

(c) Both 1 and 2

(d) Neither 1 nor 2

 

 

Answer: B

 

·        Chittaranjan Das played a prominent role in freeing India from British rule as a lawyer, politician and journalist. Netaji Subhash Chandra Bose came under the influence of Mahatma Gandhi and joined the Indian National Congress. On Gandhiji’s instructions, he started working under Deshbandhu Chittaranjan Das, whom he later acknowledged as his political guru. So, Statement 1 is correct.

·        Gopal Krishna Gokhale was also a social reformer whose goals were to promote non-violence and reform within existing government institutions. After Mahatma Gandhi’s return to India from South Africa, he joined Gokhale’s group before going on to lead the Independence movement. Gandhi regarded Gokhale as his political mentor and wrote a book in Gujarati dedicated to the leader titled ‘Dharmatma Gokhale’. Gopala Krishna Gokhale considered M.G.Ranade as his political guru. So, Statement 2 is not correct.

 

75.  Consider the following statements:

1. The history of Indian Colonialism begins with the arrival of the Portuguese in the 15th century

2. Dutch established its factories at Masulipattinam, Surat and Chinsura.

3. Portuguese captured Goa from the Vijayanagar Empire, which became the capital of the Portuguese East Indies.

Which of the statements given above is/are correct?

(a) 1 and 3 only

(b) 1 and 2 only

(c) 2 and 3 only

(d) None of the above

 

Answer: B

·        The history of Indian Colonialism begins with the arrival of the Portuguese in the 15th century, when Vasco da Gama, a Portuguese explorer, who had discovered the sea route to India at Calicut in 1498. Then they gradually started trading activities, established their forts, and factories, and even appointed Governors to look over the Portuguese control in India. and also they were the first to arrive in India and the last to leave. They established trading stations at Calicut, Cannore and Cochin. The first governor of the Portugese in India was Francis de Almeida. So, Statement 1 is correct.

·        The Dutch was the second to arrive to India after the Portugese. The Dutch founded their first factory in Masulipatnam (in Andhra) in 1605. They went on to establish trading centers in different parts of India and thus became a threat to the Portuguese. They captured Nagapatam near Madras (Chennai) from the Portuguese and made it their main stronghold in South India.

·        The Dutch established factories on the Coromandel coast, in Gujarat, Uttar Pradesh, Bengal, and Bihar. In 1609, they opened a factory in Pulicat, north of Madras. Their other principal factories in India were at Surat (1616), Bimlipatam (1641), Karaikal (1645), Chinsura (1653), Baranagar, Kasimbazar (near Murshidabad), Balasore, Patna, Nagapatam (1658) and Cochin (1663). So,Statement 2 is correct.

·        The Portuguese conquest of Goa occurred when the governor Alfonso De Albuquerque captured the city in 1510 from the Adil Shahis of Sultanate of Bijapur and not from the Vijayanagar Empire. In fact, the Sultans of Bijapur and the Vijayanagar Empire fought for their power in the Battle of Raichur in 1510, which resulted in a decisive victory for Vijayanagara forces, and the Bijapur ruler was defeated and pushed across the river, Krishna.

·        At first, Cochin was the capital but later Goa became the capital of the Portuguese East Indies. So, Statement 3 is not correct

 

76.  Consider the following statements regarding the Battle of Wandiwash:

1. It was a decisive battle in southern India between the French, under the Comte de Lally, and the British, under Sir Edward Wheeler.

2. The French were defeated by the British at Wandiwash in January 1760.

3. Post the Battle of Wandiwash, the French possessions in India were restored by the treaty of Paris (1763).

Which of the statements given above is/are correct?

(a)    1 and 2

(b)   1 and 3

(c)    2 only

(d)   2and 3

 

 

Answer:D

 

·        The Battle of Wandiwash, (1760), in the history of India, was a confrontation between the French, under the Comte De Lally, and the British, under Sir Eyre Coote (not under Edward Wheeler). It was the decisive battle in the Anglo-French struggle in southern India at Wandiwash (or Vandavasi) in Tamil Nadu during the Seven Year’s War (1756–63). So, Statement 1 is not correct.

·        This battle is the part of Third Carnatic War (1758-63), which was won by the English at Wandiwash on January 22, 1760. As a result, the Treaty of Peace of Paris (1763) was signed, it restored the French possessions in India, but the French political influence disappeared after the war. Thereafter, the French, like their Portuguese and Dutch counterparts in India, confined themselves to their small enclaves and commerce. The French were allowed to use Indian settlements for commercial purposes only and fortification of settlements was banned. So, Statements 2 and 3 are correct.

 

77.  Consider the following statements regarding the system of Dual Government in Bengal in the 18th century:

1. It was introduced by Lord Clive post the Treaty of Allahabad in 1765.

2. As the Diwan of Bengal, the English East India Company directly collected the revenue.

3. The responsibility of administration was left to Naib Diwan and Naib Nazim Muhammad Raza Khan, nominated by the company.

Which of the statements given above is/are correct?

(a) 1 and 2

(b) 2 only

(c) 2 and 3

(d) 1, 2 and 3

 

 

Answer: D

 

·       The Battle of Buxar was fought On 22 and 23 October 1764 between the forces under the

 command of the British East India Company, led by Hector Munro, and the combined armies of Mir Qasim, Nawab of Bengal till 1763, Shuja-ud-Daulah, the Nawab of Awadh, and the Mughal Emperor Shah Alam II. After the Battle of Buxar, the East India Company became the real masters of Bengal.

·       The Treaty of Allahabad was signed on August 12, 1765, between Mughal Emperor Shah Alam II

and Robert Clive as a result of the Battle of Buxar. The Treaty of Allahabad of 1765 resulted in dual administration or diarchy in Bengal.

·       Robert Clive introduced the dual system of government, i.e., the rule of the two—the Company and the Nawab—in Bengal in which both the Diwani, i.e., collecting revenues, and Nizamat, i.e., police and judicial functions, came under the control of the Company

·       The Company acquired the Diwani functions from the emperor and nizam functions from the subahdar of Bengal. The Company exercised Diwani rights as the diwan of Bengal, the English East India Company directly collected the revenue. So, Statements 1 and 2 are correct.

·       After the treaty of Allahabad, the English East India Company was made the Diwan of Bengal, but Lord Clive chose not to take over the administration of Bengal directly, and this responsibility was left to the Nawab’s Naib Diwan and Naib Nazim Muhammad Raza Khan. As naib Nazim, he was to represent the Nawab, and as naib diwan, he was to represent the Company. So, Statement 3 is correct.

 

78.  Consider the following statements regarding the Regulating Act of 1773:

1. It laid the foundations of decentralization in Indian administration.

2. It provided for the Board of Control for the Company to report on its revenue, civil and military affairs in India.

3. It was followed by the Act of Settlement of 1781 to correct its defects.

Which of the statements given above is/are correct?

(a) 1 and 3 only

(b) 2 only

(c) 2 and 3 only

(d) 3 only

 

Answer:D

·        The Regulating Act created a unified administration for India, uniting the three presidencies (Bengal, Bombay and Madras) under the authority of the Bengal’s governor, who was elevated to the new position of “Governor-General of Bengal. The Governors of Bombay and Madras presidencies were made subordinate to the Governor General of Bengal. Thus, this act laid the foundation of centralised administration in India. So, Statement 1 is not correct.

·        Regulating act of 1773 only extended the tenure of the Court of Directors from one year to four years. Their actions are supervised by the British government. The Court of Directors must report to the British parliament on the matters of Revenue, Civil and Military affairs. Whereas the Board of control was established through Pitt’s India act of 1784. So, Statement 2 is not correct.

·        The Act of Settlement was an Amending Act of 1781, which was passed by British Parliament on 5th July 1781 to remove the defects of the Regulating Act 1773. It is also known as Declaratory Act, 1781. So, Statement 3 is Correct.

 

79.  Regarding the Poligars’ Rebellion, which of the following statements is/are incorrect?

1. The Poligars were feudatories under the Maratha kingdom.

2. The oppressive land revenue system against the Poligars led to the rebellion.

Select the incorrect answer using the code given below:

(a) 1 only

(b) 2 only

(c) Both 1 and 2

(d) Neither 1 nor 2

 

Answer: A

·       The Poligars (Palayakkarar) system had evolved with the extension of Vijayanagar rule into Tamil Nadu. Each Poligar was the holder of a territory or Palayam (usually consisting of a few villages), granted to him in return for military service and tribute. They had judicial powers and dispensed justice over civil and criminal cases. So, Statement 1 is not correct.

·       The Nawab of Arcot entrusted the task of collecting land revenue arrears to the Company administration in several parts of Tamil Nadu which were controlled by Poligars under the provisions of the Karnatac Treaty of 1792. Many of the Poligars put up certain pretexts and did not pay the tribute to the British because of their oppressive land revenue system. Thus the Revolt of Poligars started. So, Statement 2 is correct.

 

80.  Which of the following was not one of the causes of the Battle of Buxar in 1764?

(a) Misuse of the farman of 1717 and the dastaks by the British for private trade.

(b) Open defiance by the deputy governor of Bihar when Mir Qasim asked for the revenue accounts of Bihar.

(c) Abolition of all duties on internal and external trade by the Nawab Mir Qasim.

(d) Oppression of the local people by the company’s servants.

 

Answer: C

·       The British acquired Bengal by the latter half of the eighteenth century. With the establishment of the British factory at Balasore in 1633, the East India Company had begun its regular trade with Bengal, and its trading ambition subsequently increased. Mughal Emperor Farrukhsiyar issued the royal Charter (Shahi Farman) in 1717.

·       The Company was permitted to issue dastaks (passes) customs-free within the province of Bengal. Dastak was the trade permit sanctioned to the East India Company by the Mughal government. Under the terms and conditions of Farrukh siyar’s Farman of 1717, the East India Company was entitled to trade in Bengal without paying the normal customs duty.

·       The Company sold dastaks at a high price to European private traders and native merchants. Consequently, the government was losing revenue on the one hand, and the native merchants were losing their business due to unequal competition with the Company and private traders on the other. The Battle was the outcome of the misuse of Farman and Dastak and also the trade expansionist aspiration of English. So, Option (a) is correct.

·       The Company thought Mir Kasim would be an ideal puppet for them. But Mir Kasim belied (failed to fulfill) the expectations of the Company. The Nawab of Bihar repeatedly requestedto submit the accounts of the revenues of Bihar to the deputy-governor of Bihar, Ram Narayan, but he did not respond. Hence, Mir Kasim couldn’t stand up to such open defiance of his authority. So, Option (b) is correct.

·       The employees of the Company misused their trade privileges. They sold their duty-free trade permits to Indian merchants, who used them to carry on duty-free trade. This deprived the Nawab of large revenues and was unfair to those local merchants who had to pay heavy duties.” To end the British’s corrupt practices, Mir Qasim abolished all duties on internal trade but not external trade. So, Option (c) is not correct.

·       After the Battle of Buxar in 1764,the East India Company started interfering in Indian affairs. So the period from 1765-72 began with the dual system of government where the Company had the authority, but no responsibility, and its Indian representatives had all the responsibility, but no authority continued for seven years.

·       This period was characterized by,

a.      The rampant corruption among servants of the Company who made full use of private trading to enrich themselves;

b.      Excessive revenue collection and oppression of peasantry;

c.      The Company’s bankruptcy, while the servants were flourishing.

·       So, Option (d) is correct.

 

81.  Consider the following pairs:

Regional kingdoms                Founders/Important ruler

1. Awadh                                    – Nizam-ul-Mulk

2. Bengal                                   – Murshid kuli khan

3. Hyderabad                            – Saadat khan

Which of the pairs given above is/are correct?

(a) 1 and 2 only

(b) 2 only

(c) 1 and 3 only

(d) 3 only

 

Answer: B

·       Burhan-ul-Mulk Sa’adat Khan was appointed subadar of Awadh in 1722 and founded a state which was one of the most important to emerge out of the break-up of the Mughal Empire. He also held the combined offices of subadari, Diwani, and faujdari. In other words, he was responsible for managing the political, financial, and military affairs of the province of Awadh. So, Pair (1) is not correct.

·       Bengal gradually broke away from Mughal control under Murshid Quli Khan, who was appointed as the naib, deputy to the governor of the province. Although never a formal subadar, Murshid Quli Khan very quickly seized all the power that went with that office. Like the rulers of Hyderabad and Awadh, he also commanded the revenue administration of the state. So, Pair (2) is correct.

·       Nizam-ul-Mulk Asaf Jah or Chin Qilich Khan, is the founder of Hyderabad state (1724-1748), and he was one of the most powerful members of the court of the Mughal Emperor Farrukh Siyar. . He was entrusted first with the governorship of Awadh, and later given charge of the Deccan. So, Pair (3) is not correct.

 

82.  Consider the following statements regarding the policies of the Company government in India:

1. The Policy of Ring-Fence of Warren Hastings created a buffer zone to defend company’s frontiers.

2. The Policy of Masterly Inactivity of John Lawrence is related to Nepal.

Which of the statements given above is/are incorrect?

(a) 1 only

(b) 2 only

(c) Both 1 and 2

(d) Neither 1 nor 2

 

Answer: B

·       So The Policy of Ring-Fence of Warren Hastings created a buffer zone to defend company’s frontiers, Statement 1 is correct.

·       Lord Lawrence, the governor-general in India during 1864-1869, initiated the policy of Masterly Inactivity. The Policy of Masterly Inactivity towards Afghanistan was pursued by the English largely as a reaction against the disastrous consequences of the first Afghan war. Hence, it is not related to Nepal. So, Statement 2 is not correct.

 

83.  With reference Battle of Plassey, consider the following statements:

1. On 23rd June 1757, Siraj-ud-Daulah was defeated by Robert Clive on the banks of the Hooghly River.

2. Immediately after the battle, Mir Jafar became the Nawab of Bengal with the support of the Jagat Seth brothers.

Which of the statements given above is/are correct?

(a) 1 only

(b) 2 only

(c) Both 1 and 2

(d) Neither 1 nor 2

 

Answer: C

·       The Battle of Plassey was a decisive victory of the British East India Company over the Nawab of Bengal and his French allies. The East India Company forces under the command of Robert Clive met the armies of Siraj-ud-Daulah on the banks of the Bhagirathi-Hooghly river on 23rd June 1757, near the small village of Plassey. Robert Clive defeated Siraj-ud-Daulah. The Battle became famous because it was the first major victory the Company won in India. So, Statement 1 is correct.

·       Robert Clive bribed Mir Jafar, the commander-in-chief of the Nawab’s army, and also promised to make him Nawab of Bengal. Jagat Seth brothers had joined hands with Robert Clive in his fight with Nawab Siraj-ud-daula. After the battle of Plassey, with the support of the Jagat Seth brothers, Mir Jafar became the Nawab of Bengal. So, Statement 2 is correct.

 

84.  Consider the following statements:

1. Non-interference in internal matters of the state.

2. Post a British resident in the court.

3. Indian rulers may maintain their own army.

4. Permanently station a British army within the territory.

Which of the above was/were the features of Subsidiary Alliance?

(a) 1 and 2 only

(b) 3 only

(c) 1, 2 and 4 only

(d) 4 only

 

Answer: C

·       Wellesley came to India with a determination to launch a forward policy to make ‘the British Empire in India’ into ‘the British Empire of India’. The system that he adopted to achieve his objective is known as the ‘Subsidiary Alliance’. It was basically a treaty between the British East India Company and the Indian princely states. The Indian state was called ‘the protected state’ and the British hereinafter were referred to as ‘the paramount power’.The paramount power should not interfere in the internal affairs of the protected state. So, Statement 1 is correct.

·       The ruler of the protected state should keep a British Resident at his court, but he should disband his own army. And he should not employ Europeans in his service without the sanction of the paramount power. So, Statement 2 is correct and Statement 3 is not correct.

·       Under the system, the allying Indian state’s ruler was compelled to accept the permanent stationing of a British force within his territory and to pay a subsidy for its maintenance. So, Statement 4 is correct.

 

85.  With reference to the Aligarh movement, consider the following statements:

1. It is a revivalist movement started by Sir Syed Ahmad Khan.

2. It opposed polygamy and promoted modern education.

3. Syed Ahmad Khan propagated his ideas through Tahdhib-ul-Akhlaq magazine.

Which of the statements given above is/are correct?

(a) 1 and 2 only

(b) 1 and 3 only

(c) 2 and 3 only

(d) 1, 2 and 3

 

Answer: C

·       The Aligarh Movement was started by Sir Syed Ahmad Khan (1817-98) for the social and educational advancement of the Muslims in India. It is a reformist movement and not revivalist movement. So, Statements 1 is not correct

·       The Aligarh movement aimed at spreading Modern education among Indian Muslims without weakening their allegiance to Islam, and social reforms among Muslims relating to purdah, polygamy, and divorce. So, Statements 2 is correct.

·       Syed’s progressive social ideas were propagated through his magazine Tahdhib-ul-Akhlaqin 1870 (Improvement of Manners and Morals).

·       The primary objective of publishing Tahzib al-akhlaq was to establishment of social harmony among the Muslims of India, revival of true Islamic traditions, remove the misconception of Islam from the masses in regards with the modern developments. So, Statement 3 is correct.

 

86.  With reference to the Revolt of 1857, consider the following statements:

1. Immediate cause of the revolt of 1857 was the replacement of the Brown Bess rifle with the Enfield rifle.

2. General Service Enlistment Act of 1856 was one of the reasons for the discontent among sepoys.

3. Frequent use of the quo warrant by the company to the landed aristocracy.

Which of the above are considered as the major causes of the revolt?

(a) 1 and 2 only

(b) 2 and 3 only

(c) 1 and 3 only

(d) 1, 2 and 3

 

Answer: D

·       The revolt of 1857 started on 10th May when the Company’s Indian soldiers at Meerut rebelled. It was called the Sepoy Mutiny by the British, it is now recognized as the First War of Independence against the British rulers. The immediate cause of this is, the introduction of greased cartridges with the Enfield Rifles in 1856 by the government by replacing the old-fashioned musket, Brown Bess guns. The loading process of the Enfield rifle involved bringing the cartridge to the mouth and biting off the top. There was a rumour among the Sepoys in January 1857 that the greased cartridge contained the fat of cow and pig, which caused the fire to revolt. So, Statement 1 is correct.

·       At that time, the religious belief of Hindus was that the travel across the sea was forbidden and led to loss of caste. But in 1856, Lord Canning’s government passed the General Service Enlistment Act which decreed that all future recruits to the Bengal Army would have to give an undertaking to serve anywhere their services might be required by the government (both within and outside India) and this caused resentment among the sepoys. So, Statement 2 is correct.

·       One of the main economic ca uses that aided the revolt is the issue faced by Zamindars, who are the traditional landed aristocracy, often saw their land rights forfeited with frequent use of a quo warrant by the British administration and as a result, they lost their status in the villages which created the discontent among them against the British. So, Statement 3 is correct.

 

87.  Consider the following statements with reference to East India Association:

1. East India association was organized by Dadabhai Naoroji at Calcutta in 1866.

2. Members of the East India Association were Indians and retired British officials.

3. Dadabhai Naoroji was the first president of the East India association.

Which of the statements given above is/are correct?

(a) 1 only

(b) 2 only

(c) 3 only

(d) 1 and 3 only

 

Answer:B

·        The East India Association was founded by Dadabai Naoroji in 1866, in collaboration with Indians and retired British officials in London (not in Calcutta).

·        It superseded the London India Society and was a platform for discussing matters and ideas about India, and providing representation for Indians to the Government.

·        Naoroji delivered the first lecture to the Association on 2 May 1867.

·        The Association’s first President was Lord Lyveden and not Dadabai Naoroji. So, Option (b) is correct.

 

88.  During the last half of the 18th century and the first half of the 19th century, major battles were fought between Maratha and the Britishers. Identify the governor-general during the major Anglo- Maratha war:

1. Warren Hasting

2. Lord Cornwallis

3. Marques Hasting

4. Lord Wellesley

Select the correct answer using the codes given below:

(a) 1, 3 and 4 only

(b) All of the above

(c) 2 and 3 only

(d) 1 and 2 only

 

Answer: A

·       The Governor-Generals associated with the Anglo-Maratha wars are as follows:

·       Warren Hastings was involved in the First Maratha War which happened in 1775-82 and signed the Treaty of Salbai in 1782

·       Lord Wellesley is associated with the Second Maratha War which happened in 1803-05.

·       Lord Hastings or Marques Hastings is engaged in the Third Maratha War (1817-19) where the dissolution of the Maratha Confederacy and the creation of the Bombay Presidency (1818) happened.

·       Lord Cornwallis was not associated with Anglo Maratha Wars. But he was associated with Third Anglo Mysore War. So, Option (A) is correct.

 

 

 

89.  Brahmo samaj was founded by Raja Ram Mohan Roy for reform in Indian Society. Consider the following statement related to the Brahmo Samaj:

1. The samaj opposed idol or image worship.

2. The rationalist perspective of Brahmo Samaj led to the repudiation of the infallibility of the Vedas.

3. Brahmo Samaj incorporated the teachings of other religions too.

Which of the statement given above is/are correct?

(a) 3 only

(b) 1 and 2 only

(c) 2 and 3 only

(d) 1, 2 and 3

 

Answer:D

·       Brahmo Sabhaor “One God Society” was founded by Raja Rammohan Roy in 1828 and later, it was known as Brahmo Samaj. It was against idol or image worship, and it did not allow animal sacrifices or offerings. So, Statement 1 is correct.

·       The Brahmo Samaj refused to accept the authorityof the Vedas and they wanted to reform the same. They used a rational approach to study tradition and they evaluated the contemporary socio-religious practices from the standpoint of social utility and to replace faith with rationality. As a consequence, the infallibility of the Vedas was repudiated by them. The long-term agenda of the Brahmo Samaj was to purify Hinduism and preach monotheism. So, Statement 2 is correct.

·       It was committed in opposition to idolatry and meaningless rituals of Hindus and adopted some Christian practices in its worship. It is influenced by Islam and Christianity and denounces polytheism (The belief in more than one God).The Samaj also tried to incorporate teachings of other religions and kept its emphasis on human dignity and criticism of social evils such as Sati Pratha. So, Statement 3 is correct.

 

90.  Theosophical Society was established by Madame H. P. Blavatsky and Colonel M. S. Olcott in New York, United States of America. In this context, consider the following statement with reference to the Theosophical Society.

1. The Society believed in reincarnation and Karma.

2. It’s philosophical impact was limited to the small segment of the westernized class.

3. Members of the Theosophical Society actively took part in the Home Rule movement.

Which of the statement given above are not correct?

(a) 1 and 2 only

(b) 2 and 3 only

(c) 1 and 3 only

(d) None of the above

 

Answer: D

·       Some Theosophical Ideas which believed by the Theosophical society includes, “The cycle of reincarnation is ruled by the law of cause and effect.” And “…whatever we sow, we will inevitably reap. This is the law of karma by which we weave our own destiny through the ages.”. Hence, Theosophical society believed in reincarnation and Karma. So, Statement 1 is correct.

·       One of the main objectives of the theosophical society was to advocate the revival and strengthening of the ancient religions and philosophies which were mostly conservative in nature. In general, it lacked the support of the westernised class of people as most of them were rational in thinking. So, it was able to have impact only on a small segment of westernised class. So, Statement 2 is correct.

·       Annie Besant became the second President of the Theosophical Society which had its headquarters in Madras. She became a theosophist after coming under the influence of Madame Blavatsky of Theosophical Society. Also being the President of Theosophical society. Annie Besant started the home rule league in 1916, with the aim of trying to organise people for the demand of home rule for India, in which the members of theosophical society also actively take part in the Home Rule movement. So, Statement 3 is correct.

 

91.  The British Government reformed some of the social practices in Indian Society. Consider the following statement related to the social reforms done by the British administrator:

1. Lord Ellenborough supported the passing of a law against buying and selling of slaves or people as commodities.

2. The cruel practice of Human sacrifice was suppressed by Lord Harding I.

Which of the statement given above is/are correct?

(a) 1 only

(b) 2 only

(c) Both 1 and 2

(d) Neither 1 nor 2

 

Answer: C

·       Slavery as a system of labour exploitation was prevalent in British India, where the slave labours were used for the purpose of fulfilling the enormous labour requirement arose due to the introduction of plantation cropsand slope cultivation in Ceylon, Mauritius,Fiji, Malaya, the Caribbean islands, Nataland South Africa. This practice of slavery was abolished in 1843, during the reign of Lord Ellenborough (1842-1844).

·       Under the Indian Slavery Act of 1843, the sale of any person as a slave was banned. Anyone buying orselling enslaved people would be booked under the Indian Penal Code with an offence carrying strict punishment. So, Statement 1 is correct.

·       During the period, Lord Hardinge I (1844-1848), the governor-general, made social reforms, including abolishing female infanticide, and suppression of the practice of human sacrifice among the Gonds of central India. So, Statement 2 is correct.

 

 

 

 

92.  Which of the following statements regarding the land revenue policies of the English East India Company is/are incorrect?

1. Cornwallis introduced Permanent Settlement hoping that the rule of law and private property rights would bring in modernization to the economy and Society.

2. Munro introduced Ryotwari Settlement with the intention of preserving India’s village communities by emphasizing the protective role of the Company’s Government.

Select the incorrect answer using the code given below:

(a) 1 only

(b) 2 only

(c) Both 1 and 2

(d) Neither 1 nor 2

 

Answer: D

·       Before 1793, land ownership was based on community, and there was no concept of individual ownership of land. But Lord Cornwallis introduced Permanent Settlement, which created individual property rights over land. He hoped that the rule of law and private property rights would liberate individual enterprises from the shackles of custom and tradition. And it would increase investment in agricultural land and bring modernization to the economy and society. So, Statement 1 is correct.

·       Sir Thomas Munro, governor of Madras, thought that the Cornwallis permanent settlement system did not heed Indian tradition and experience. He thought such reforms had to be modified to suit the Indian context and so he introduce the Ryotwari Settlement to preserve India’s village communities and emphasize the protective role of the company’s government. But ultimately, he aimed to consolidate the Company’s state in the south by expanding its revenue base, where land taxes would be collected directly from the peasants by a large number of British officers. So, Statement 2 is correct.

 

93.  The term “abwabs” in British India referred to:

(a) Revenue collected by Mansabdars from their Jagirs.

(b) Revenue paid by the peasant as extra-legal charges to the zamindar.

(c) Revenue paid by the peasant to revenue farmers at the rates decided by the British.

(d) Revenue paid by the smaller zamindars to the British through the bigger zamindars.

 

Answer: B

·       Under the Permanent Settlement system, the nawabs were used to collect revenue from the zamindars. Some of them were big landlords who controlled large areas and had their own armed retainers and others were smaller zamindars, who paid revenue either directly to the state or through the bigger zamindars.

·       Peasants undertook cultivation and paid the zamindars at customary rates, which often varied from subdivision to subdivision, and also sometimes extralegal charges called abwabs were collected from them by the zamindars. Abwabs are the cesses collected from the peasants in addition to land revenues. So, Option (b) is correct.

 

94.  What are the major reasons for the decline of the Portuguese in India?

1. Defective governance system

2. Good trade practices compared to other European powers

3. Rise of Vijayanagara empire.

4. Religious policies like conversion to Christianity

Choose the correct answer from the options given below:

(a) 1,2 and 3 only

(b) 2 and 3 only

(c) 1 and 4 only

(d) 1, 3 and 4 only

 

Answer: C

·       After Alfonzo-De-Albuquerque, the Portuguese viceroy of India, their successors were ineffective, making the Portuguese administration in India weak. The Portuguese power in India declined as new European trading rivals posed a challenge. The arrival of the Dutch, French, and British in India also ensured the decline of Portuguese power. So, Statement 1 is correct.

·       Portuguese were the first European country to re-establish a direct trade link with India. The arrival of other European rivals and their dishonest trade practices weakened their activities in India. So, Statement 2 is not correct

·       The Portuguese had good relations with the Vijayanagara Empire. Thus, the rise of the Vijayanagar Empire favored the growth of Portuguese power in India, and its decline reduced the influence of the Portuguese in India. After the Battle of Talikotta in 1565, the Vijayanagar Empire started to decline, substantially affecting Portuguese influence in India. So, Statement 3 is not correct.

·       The religious policies of the Portuguese, such as the activities of the Jesuits, gave rise to political fears. Their antagonism toward the Muslims and the Portuguese conversion policy to Christianity dissatisfied Hindus, resulting in the decline of Portuguese power in India. So, Statement 4 is correct.

 

95.  With reference to Alfonso de Albuquerque, consider the following statements:

1. He is the real founder of Portuguese in the East.

2. He abolished the practice of Dowry in Goa.

3. He introduced a system of permits for non-Portuguese ships.

Which of the statements given above is/are correct?

(a) 1 and 2 only

(b) 2 and 3 only

(c) 1 and 3 only

(d) 1, 2 and 3

 

Answer: C

·       The first Portuguese to land in India was Vasco De Gama in 1498. After Almeida, Albuquerque became the second Viceroy of the Portuguese in India. He captured Goa from the Sultan of Bijapur in 1510 and made it the capital. He also strengthened his relationship with the Vijayanagar Empire. He can be rightly called the real founder of the Portuguese power in India. So, Statement 1 is correct.

·       The practice of Sati was first banned in Goa in 1515 by the Portuguese governor Albuquerque. Later in 1829, Lord William Bentick, through the Bengal Sati Regulation, banned sati practice and not Dowry in all jurisdictions of British India. So, Statement 2 is incorrect

·       Cartaz was a naval trade license issued by the Portuguese in the Indian Ocean during the sixteenth century under the rule of the Portuguese empire. It shared similarities with the British Navicert system of 1939-45.

·       The Cartaz system required non-Portuguese ships to visit parts controlled by Portuguese purchase permits by paying a fee before venturing into the sea. If any ship entered the water without a permit or pass, ships were seized and goods confiscated. The Cartaz system was introduced by Francisco De Almeida.

·       Albuquerque secured for Portugal the strategic control of the Indian Ocean by establishing bases overlooking all the entrances to the sea. There were Portuguese strongholds in East Africa, off the Red Sea, at Ormuz; in Malabar; and at Malacca. The Portuguese, under Albuquerque bolstered their stranglehold by introducing a permit system for other ships and exercising control over the major shipbuilding centres in the region. So, Statement 3 is correct.

 

96.  The Kundara Proclamation with respect to Indian history is related to:

(a) Velu Thampi’s call to openly take up arms against the British.

(b) Wazir Ali’s guards attacking Europeans and the Magistrate of Benares.

(c) Lakshaman Dawa’s call to Bundelkhand soldiers to protect the Ajaygarh Fort against British siege.

(d) Mufti Muhammad Aiwaz’s call to his followers to revolt against the imposition of police tax.

 

Answer: A

·       The East India Company’s harsh conditions imposed on the state of Travancore after both of them agreed to a subsidiary alliance arrangement under Wellesley in 1805 caused deep resentment in the region.

·       The ruler was not able to pay the subsidy and fell in arrears. The British resident of Travancore was meddling in the state’s internal affairs.The high-handed attitude of the Company compelled Prime Minister (or Dalawa) Velu Thampi to rise against the Company, assisted by the Nair troops.

·       Velu Thampi addressed a gathering in Kundara, openly calling for taking up arms against the British to oust them from the native soil. This was later known as the Kundara Proclamation. There was a large-scale rebellion against the British as a result. A large military operation had to be undertaken to restore peace. The Maharaja of Travancore had not wholly supported the rebellion and defected to the side of the Company. Velu Thampi killed himself to avoid capture. The rebellion petered out. So, Option (a) is correct.

 

97.  Which of the following is/are correct with reference to the Indian National Social Conference?

1. Its first session was held in Calcutta in December 1887.

2. It was founded by M.G.Ranade and Anand Mohan Bose.

3. The conference opposed kulinism and polygamy.

Select the correct answer using the codes given below:

(a) 1 and 2 only

(b) 2 only

(c) 1 and 3 only

(d) 3 only

 

Answer: D

·       M.G. Ranade and Raghunath Rao founded the Indian Social Conference. It met annually from its first session in Madras in 1887 at the same time and venue as the Indian National Congress.

·       It focused on the social issues of importance; it could be called the social reform cell of the Indian National Congress.

·       The conference advocated inter-caste marriages but opposed the practices of Polygamy and kulinism.

·       Kulinism is a custom that Raja Ballala Sena of Bengal introduced, which asserted social and

·       religious superiority over others. Polygamy is a type of relationship typically involving a person marrying more than one partner. Polygamy is the opposite of monogamy, where one person marries one spouse.

·       Indian Social Conference launched the ‘Pledge Movement’ to inspire people to take a pledge against child marriage.So, Option (d) is correct.

 

98.  The Mahalwari system is one of the three main land revenue systems in British India. Consider the following statement related to the Mahalwari system:

1. Mahalwari system was introduced by Holt Mackenzie in the North-Western province of Bengal.

2. Every individual farmer had their share in fixed revenue for each mahal, and revenue was collected by the village headmen.

3. Governor-general Lord Canning was responsible for introducing Mahalwari in Punjab and Awadh.

Which of the statements given above is/are correct?

(a) 1 only

(b) 1 and 2 only

(c) 2 and 3 only

(d) 1, 2 and 3

 

Answer: B

·       In the North Western Provinces of the Bengal Presidency (most of this area is now in Uttar Pradesh), an Englishman called Holt Mackenzie devised the new system which came into effect in 1822. Under this system, the state made settlements with either the village community or, in some cases, the traditional ‘taluqdar’. Each such fiscal unit was called a ‘mahal’. Under this system, some recognition was given to collective proprietary rights. It came to be known as Mahalwari system. So, Statement 1 is correct.

·       Under the Mahalwari system, the charge of collecting the revenue and paying it to the Company was given to the village headman on behalf of the whole village rather than the zamindar. The revenue under the Mahalwari system was to be revised periodically and not fixed permanently. So, Statement 2 is correct.

·       Even though Holt Mackenzie devised it in 1822, it was later popularised by Lord William Bentick in 1833 in Agra and Awadh and extended to Madhya Pradesh and Punjab. So, Statement 3 is not correct.

 

99.  Consider the following statements regarding Mahadji Scindhia of Gwalior under the Maratha empire:

1. The Mughal king Shah Alam II made him the amir al-umara (“head of the amirs”) and naib wakii-I mutlaq (“deputy regent”) of his affairs.

2. He was defeated by the British and forced by a treaty in 1803 to surrender his territories to the north.

3. He employed a large number of European soldiers in his force which made him an effective military commander.

Which of the statements given above is/are correct?

(a) 1 and 2

(b) 1 and 3

(c) 2 only

(d) 2 and 3

 

Answer: B

·       Sindhia are the Maratha ruling family of Gwalior, which dominated the politics of northern India during the 18th century. Mahadaji, Sindhia (1761-94) was an effective and innovative military commander. He employed a large number of European soldiers in his force and made him an effective military commander. His power grew rapidly after 1770.

·       The Mughal king Shah Alam II made him the “deputy regent” of his affairs in the mid-1780s And he zealously guarded the privileges and titles granted to him by Shah Alam, such as amir alumara (“head of the amirs”) and na’ib wakii-i mutlaq (“deputy regent”).So, Statement 1 and 3 Is Correct.

·       Maratha chief Daulat Rao Sindhia (1794-1827), who was defeated by the British and forced to sign Treaty of Surji-Arjungaon in 1803 and also surrendered their territories including the Ganges-Yamuna doab (territory between the rivers), Agra, and Sindhia’s territories in Gohad and Gujarat were entrusted to the British East India Company.thus,Treaty in 1803 was signed by Maratha chief Daulat Rao Sindhia not by Mahadaji,Sindhia. So, Statement 2 is not Correct.

 

100.         Who among the following were the founders of Sadharan Brahmo Samaj?

1. Anand Mohan Bose

2. Umesh Chandra Dutta

3. Keshab Chandra Sen

Choose the correct answer using the options given below:

(a) 3 only

(b) 1 and 3 only

(c) 1 and 2 only

(d) 1, 2 and 3

 

Answer: C

·        The followers of KC Sen started another samaj called Sadhran Brahmo Samaj, mainly due to a disagreement with him.

·        Raja ram Mohan Roy founded Brahma Samaj.

·        Keshab Chandra sen joined Brahma Samaj in 1858, and he was instrumental in popularising the movement, and its branches were opened outside Bengal.

·        In 1885, he was dismissed from the office of acharya because of a conflict of interest between K C sen and Debendranath Tagore.

·        Later in 1866, K C Sen, along with his followers, started the Brahmo Samaj of India

·        After 1878, the followers of Keshab set up a new organization called the Sadharan Brahmo Samaj due to a disagreement with him about Keshab’s inexplicable act of getting his thirteen year- old daughter married to the minor Hindu Maharaja of Cooch-Behar with all the orthodox Hindu rituals.

·        It was started by Ananda Mohan Bose, Shibchandra Deb and Umesh Chandra Datta, not by  K C Sen. So, Option (c) is correct.

 

Leave a Reply

Your email address will not be published. Required fields are marked *

Back to top button